Sie sind auf Seite 1von 154

Contenido

Cap tulo 1. 2. 1. 3. 1.

Induccin Matemtica o a

5 5 11 16 21 22 27 31 33 39 45 45 68 72 89 95 96 100 105 117 118 1

Principio de induccin matemtica o a El teorema del binomio Induccin matemtica generalizada o a Matrices Producto de matrices Suma de matrices Sistemas de ecuaciones El mtodo de Gauss e Sistemas de ecuaciones lineales generales Determinantes Semntica geomtrica a e Caso general Propiedades de los determinantes Regla de Cramer Nmeros complejos u Suma y producto Restas y divisiones Representacin geomtrica o e

Cap tulo 2. 2. 1.

Cap tulo 3. 2.

Cap tulo 4. 1. 2.

4.

3.

Cap tulo 5. 2. 1.

3.

Cap tulo 6. Polinomios 1. Algebra

Contenido

3.

2. 4.

Algoritmo de la divisin o Divisibilidad Mximo comn divisor a u El teorema fundamental del algebra Acotacin de ra o ces Acotacin de ra negativas o ces Ra enteras ces Ra racionales ces

119 123 125 129 137 140 141 146 151 153

5. 7.

6.

8.

9.

Bibliograf a Indice

Notas de Algebra Superior Csar Bautista Ramos e Facultad de Ciencias de la Computacin o Benemrita Universidad Autnoma de Puebla e o

Cap tulo 1

Induccin Matemtica o a

1. Principio de induccin matemtica o a


El llamado principio de induccin matemtica es una tcnica de demostracin o a e o que a veces puede usarse para demostrar propiedades que dependen de los nmeros naturales. u Supngase que P (n) es una proposicin que depende de n N. Si se quiere o o demostrar una propiedad del tipo n N, P (n). lo que se podr hacer es usar el llamado principio de induccin matemtica. a o a Este consiste de los siguientes pasos caso base:: demostrar la validez de P (n) cuando n = 1; hiptesis de induccin:: se supone como cierta P (n); o o paso inductivo:: demostrar P (n + 1). Ejemplo 1. Demostrar que n N, 1 + 2 + + n = Demostracin. Caso base: si n = 1: o 1 (1 + 1) 1= 2 es cierta. hiptesis de induccin: supongamos que o o 1 + 2 + + n = n(n + 1) 2 (1) 5 n(n + 1) 2

1. Induccin Matemtica o a

paso inductivo: tenemos que demostrar (n + 1)(n + 2) . 2 En efecto; desarrollemos el lado izquierdo de la ecuacin (2) o 1 + 2 + + (n + 1) = 1 + 2 + 3 + + (n + 1) = (1 + 2 + 3 + + n) + (n + 1) = (2)

n(n + 1) + (n + 1), por hiptesis de induccin, o o 2 n(n + 1) + 2(n + 1) = 2 (n + 1)(n + 2) = , factorizando. 2

El proceso de induccin matemtica es muy frecuente (y particularmente o a importante para la teor de la computacin). Por ejemplo, est impl a o a cito en la denicin siguiente: o Denicin 2 (Factorial). Sea n N. Se dene o (1) 0! = 1; (2) n! = n (n 1)!, si n 1. La notacin n! se lee factorial de n. Obsrvese, que en particular o e (n + 1)! = (n + 1)n! Ejemplo 3. Calculemos el factorial de 4: 4! = 4 3!, seg n ii) de la denicin, u o = 4 3 2 1!, una vez ms por ii), a = 4 3 2!, de nuevo seg n ii), u (3)

= 4 3 2 1 0!, por ii), = 24.

= 4 3 2 1 1, usando i) de la denicin, o

Como puede notarse en el ejemplo anterior, el proceso anterior termina gracias a que se deni 0! no como una nueva frmula, sino como el valor jo o o 1. Si no hubieramos denido 0 factorial, el proceso anterior no terminar nunca. a En general, esta es la razn de ser del caso base, en el proceso de induccin o o matemtica. Por lo que no es extrao que el caso base no siempre comience en a n n = 1. A veces, como en el ejemplo siguiente, se puede comenzar la induccin o desde otros nmeros. u

1. Induccin matemtica o a

Ejemplo 4. Probar que n 4, 2n < n! Demostracin. Caso base: n = 4 o 24 < 4!


16 24

es cierto. Hiptesis de induccin: Supongamos cierto o o 2n < n! Paso inductivo: Por demostrar que 2n+1 < (n + 1)! Como 1 < n (de hecho aqu 4 n, por la hiptesis general (no con o fundir con la hiptesis de induccin), entonces 2 < n + 1, desigualdad que o o multiplicamos lado a lado con la desigualdad (4) para obtener siendo el lado derecho (n + 1)n! = (n + 1)!, usando la denicin del factorial o de n + 1 (ver ecuacin (3)). Hemos obtenido o 2n+1 < (n + 1)! 2 2n < (n + 1)n! (4)

Propiedad 1. n N, n2 + n es par. Demostracin. Caso base: n = 1: 12 + 1 es par, es cierto. o Hiptesis de induccin: Supongamos que o o n2 + n es par. Paso inductivo: por demostrar que (n + 1)2 + (n + 1) es par. En efecto, (n + 1)2 + (n + 1) = (n2 + 2n + 1) + (n + 1) = (n2 + n) + (2n + 2)
par par

donde el primer sumando es par, seg n la hiptesis de induccin. Luego, u o o como suma de pares es par, se obtiene que (n + 1) 2 + (n + 1) es par. Propiedad 2. n N, n3 n es mltiplo de 6. u

1. Induccin Matemtica o a

Demostracin. Caso base: si n = 1: o es cierto. 13 1 = 0 es m ltiplo de 6. u

Hiptesis de induccin: Supongamos o o n3 n es m ltiplo de6 u Paso inductivo: por demostrar que En efecto, (n + 1)3 (n + 1) es m ltiplo de 6. u

(n + 1)3 (n + 1) = n3 + 3n2 + 3n + 1 n 1
mltiplo de 6 u par

o = (n3 n) +3 (n2 + n), hip. de induccin y prop. 1

= 6u + 3(2s), los m ltiplos de 6 son de la forma 6u, u = 6(u + 6) siendo este ultimo un m ltiplo de 6. u Observemos el siguiente fenmeno: o 1 = 12 ; 1 + 3 = 22 ; 1 + 3 + 5 = 32 ; 1 + 3 + 5 + 7 = 42 ; 1 + 3 + 5 + 7 + 9 = 52 ; uno podr inducir que la siguiente linea debe de ser a 1 + 3 + 5 + 7 + 9 + 11 = 62 como en efecto lo es. Tambin uno puede inducir cual deber ser la linea 666: e a pero, cul debe de ser el ultimo sumando del lado izquierdo? Este puede a inferirse de observar el ultimo sumando de las primeras 5 ecuaciones. Notemos que 11 32 1 + 3 + 5 + +? = 6662

53

11 6

79

1. Induccin matemtica o a

lo cual sugiere la relacin: o k o equivalentemente pues m = (k + 1)/2 k = 2m 1. Por tanto, podemos inferir que 1 + 3 + 5 + + 1331 = 6662 como puede demostrarse con una calculadora. Sin embargo puede demostrarse en general: Propiedad 3. n N, 1 + 3 + + (2n 1) = n2 . Demostracin. Caso base: n = 1: o 1 = 12 es cierto. Hiptesis de induccin: Supongamos que o o n N, 1 + 3 + + (2n 1) = n2 Paso inductivo: por demostrar que n N, 1 + 3 + + (2(n + 1) 1) = (n + 1)2
2n+1

(k + 1) 2

(2m 1) m

En efecto: 1 + 3 + 5 + + (2(n + 1) + 1) = 1 + 3 + 5 + + (2n + 1) + (2(n + 1) 1) = n2 + 2n + 1 = (n + 1)2

= n2 + (2(n + 1) 1), por hiptesis de induccin o o

Para no cansar al lector, en lo sucesivo evitaremos distiguir, de manera tan redundante (en negrillas) el caso base, la hipesis de induccin y el paso o o inductivo. Propiedad 4. Para cualquier n N 1 1 n 1 + + + = 12 23 n(n + 1) (n + 1)

10

1. Induccin Matemtica o a

Demostracin. Por induccin matemtica sobre n. Si n = 1 se obtiene o o a 1 1 = 12 1+1

lo cual es cierto. Supongamos cierta la ecuacin del enunciado de la o propiedad. Tenemos que demostrar que 1 1 1 n+1 + + ... = 12 23 (n + 1)(n + 2) n+2 para lo cual desarrollamos el lado izquierdo. 1 1 1 + + ... + 12 23 (n + 1)(n + 2) 1 1 1 1 = + + ... + + 12 23 n(n + 1) (n + 1)(n + 2) n 1 = + , por hiptesis de induccin o o n + 1 (n + 1)(n + 2) 1 (n + 2)n + = (n + 2)(n + 1) (n + 1)(n + 2) n(n + 2) + 1 = (n + 1)(n + 2) n2 + 2n + 1 = (n + 1)(n + 2) (n + 1)2 = (n + 1)(n + 2) n+1 = n+2 Tarea 1. Demostrar que n N se cumplen: (1) 2 + 4 + 6 + + 2n = n(n + 1) (2) 1 5 + 2 9 + + n(4n + 1) = n(n + 1)(8n + 7)/6 (4) n2 (n + 1)2 4

(3) 1 2 + 2 3 + 3 4 + + n(n + 1) = n(n + 1)(n + 2)/3 13 + 2 3 + + n 3 = (5) (6)

1 1 2n 1 1 + 2 + + n = 2 2 2 2n 1 1 n 1 + + + = 14 47 (3n 2)(3n + 1) 3n + 1

2. El teorema del binomio

11

(7) Si q = 1, 1 + q + q2 + + qn = q n+1 1 q1

2. El teorema del binomio


Denicin 5 (Binomial). Sean n, m nmeros enteros no negativos tales que o u n m 0. Se dene el binomial de n en m como n m Ejemplos 6. (1) 1 1 (2) 2 1 (3) 3 2 (4) n 0 (5) n 1 = = n! =1 0!(n 0)! = 3! =3 2!1! = 2! =2 1!1! = 1! =1 1!0! = n! m!(n m)!

n! n(n 1)! = =n 1!(n 1)! (n 1)!

Los binomiales son los nmeros que aparecen en el siguiente arreglo, llamado u tringulo de Pascal: a 1 1 1 1 1 4 3 6 . . . 2 3 4 1 1 1 1

12

1. Induccin Matemtica o a

esto es, el mismo tringulo puede escribirse como a

0 0 1 0 2 0 3 0 4 0 4 1 3 1 4 2 . . . 2 1 3 2 4 3 1 1 2 2 3 3 4 4

Algunas de las propiedades de propiedades de los nmeros naturales se u pueden demostrar por induccin matemtica. Pero esto no signica que uno o a est obligado a usar tal principio. Por ejemplo, el teorema de Pascal se puede a demostrar directamente sin usar ninguna clase de induccin matemtica. o a

Teorema 1 (Pascal). Si n, m N tales que n 1 m, entonces

n1 n1 + m m1

n m

2. El teorema del binomio

13

Demostracin. o n1 n1 + m1 m

= = = = = = = = = =

(n 1)! (n 1)! + m!(n 1 m)! (m 1)!(n 1 m + 1)! (n 1)! (n 1)! + m!(n 1 m)! (m 1)!(n m)! (n 1)!(n m) m(n 1)! + m!(n m 1)!(n m) m(m 1)!(n m)! (n 1)!(n m) m(n 1)! + m!(n m 1)!(n m) m(m 1)!(n m)! (n 1)!(n m) m(n 1)! , por n! = n(n 1)! + m!(n m)! m!(n m)! (n 1)!(n m) + m(n 1)! m!(n m)! (n 1)!(n m + m) , factorizando, m!(n m)! n(n 1)! m!(n m)! n! , pues n! = n(n 1)! m!(n m)! n m

Denicin 7 (Sumatoria). Sean c0 , c1 , . . . , cn R. Se dene la sumatoria o desde i=0 hasta n como


n i=0

ci = c 0 + c 1 + c 2 + + c n

Ejemplos 8. (1) (2) (3) (4)


2 4 i=0 i = 0 + 1 + 2 + 3 5 2 2 2 2 i=1 i = 1 + 2 + 3 5 i=2

+ 4 = 10 + 42 + 52 = 55 = 56

6 i

6 6 6 6 + + + 2 3 4 5
2

(x + y) =
i=0

2 2i i x y i

Propiedad 5. (1)
n i=0 ci

n j=0 ci

14

1. Induccin Matemtica o a

(2) (3) a (4)

n+1 n i=0 ci = i=0 ci + cn+1 n i=0 ci = i=0 aci n n i=0 (ci + di ) = i=0 ci +

n i=0 di

Demostracin. o (1) Las sumatorias coinciden porque ambas producen la misma suma:
n i=0

ci = c 0 + c 1 + + c n

n j=0

cj = c 0 + c 1 + + c n

(2)
n+1 i=0

ci = c0 + c1 + + cn + cn+1 = (c0 + c1 + + cn ) + cn+1


n

=
i=0

+cn+1

(3)
n

a
i=0

ci = a(c0 + c1 + + cn ) = ac0 + ac1 + + acn


n

=
i=0

aci

(4)
n i=0

(ci + di ) = (c0 + d0 ) + (c1 + d1 ) + + (cn + dn ) = (c0 + c1 + + cn ) + (d0 + d1 + + dn )


n n

=
i=0

ci +
i=0

di

Ejemplo 9. Probar que


n

n 1 N,

i3 =
i=1

n(n + 1) 2

2. El teorema del binomio

15

Demostracin. Por induccin matemtica sobre n. Si n = 1: o o a


1

i3 = 1 3 =
i=1

1(1 + 1) 2

Directamente examinemos el paso inductivo. Tenemos que demostrar que


n+1

i3 =
i=1

(n + 1)(n + 2) 2

Calculemos:
n+1 n

i =
i=1 i=1

i3 + (n + 1)3 n(n + 1) 2 + (n + 1)3 , por hiptesis de induccin, o o 2 n2 (n + 1)2 4(n + 1)3 + 4 4 2 (n2 + 4(n + 1)) (n + 1) 4 2 (n2 + 4n + 4) (n + 1) 4 (n + 1)2 (n + 2)2 4 (n + 1)(n + 2) 2 2

= = = = = =

Tarea 2. Probar que (1) (2) (3)


n i n+1 1)/4; i=0 5 = (5 n i=1 i/(i + 1) = n/(n + 1); n 2 i=0 i = n(n + 1)(2n + 1)/6.

Teorema 2 (binomio de Newton). Sean a = 0, b = 0 R, n 1 N. Entonces n n ni i (a + b)n = a b (5) i


i=0

Demostracin. Por induccin matemtica sobre n. Si n = 1, o o a


1 i=0

1 1i i a b = i

1 1 0 1 0 1 a b + a b = a + b = (a + b)1 0 1

16

1. Induccin Matemtica o a

Supongamos que (5) se cumple. Veriquemos el paso inductivo: (a + b)n+1 = (a + b)n (a + b)


n

=
i=0 n

n ni i a b (a + b), por hiptesis de induccin, o o i n (an+1i bi + ani bi+1 ) i n n+1i i a b + i


n i=0

=
i=0 n

=
i=0

n ni i+1 a b i

n n n1 2 n n n n+1 abn a b + + a b+ a + n 2 1 0 +
n

n 0 n+1 n n n1 2 n n a b abn + a b + + a b+ n n1 1 0 n n + i i1 an+1i bi + bn+1

=a

n+1

+
i=1 n

= an+1 +
i=1

n + 1 n+1i i a b + bn+1 , por el teorema de Pascal, i


n i=1

n + 1 n+1 a + 0
n+1

n + 1 n+1i i n + 1 n+1 a b + b i n+1

=
i=0

n + 1 n+1i i a b i

3. Induccin matemtica generalizada o a


Existe otra forma de la induccin matemtica (de hecho equivalente) que se o a llama el principio de induccin generalizado. Supongamos que queremos deo mostrar una proposicin del tipo o n N, P (n). El principio de induccin generalizado dice que podemos hacer los siguientes o pasos: Caso base: Probar, para un k0 jo, P (1), P (2), . . . , P (k0 ). Hiptesis de induccin: Suponer que P (j) es cierto para cualquier o o k0 j n.

3. Induccin matemtica generalizada o a

17

Paso inductivo: Probar P (n + 1). Para ejemplicar haremos uso de los nmeros de Fibonacci. Estos de deu nen de manera recursiva, similarmente a los factoriales. Denicin 10. Los nmeros de Fibonacci, denotados F 0 , F1 , . . . , Fn , . . ., se o u denen como Fn = n, Fn1 + Fn2 , si n = 0 n = 1; si n 2

Esto es: F0 = 0, F1 = 1, F2 = F1 + F0 = 1, F3 = F2 + F1 = 1 + 1 = 2, F4 = F3 + F2 = 2 + 1 = 3, etc. Propiedad 6. Los nmeros de Fibonacci satisfacen u


n

Fi2 = Fn Fn+1 ,
i=1

n 1 N.

(6)

Demostracin. Por induccin matemtica sobre n. o o a Caso base: Vericamos para n = 1 (en este caso k 0 = 1 en el caso base del principio de induccin generalizado). Tenemos que o 1 Fi2 = F1 = 1, mientras que F1 F2 = 1. Es decir, se cumple i=1
1

Fi = F 1 F2
i=1

Hiptesis de induccin: Suponemos que es cierto que o o


j i=1

Fi2 = Fj Fj+1 , para 1 j n

Paso inductivo: por demostrar que


n+1 i=1

Fi = Fn+1 Fn+2 En efecto: tenemos que


n+1 n

(7)

Fi2
i=1

=
i=1

2 Fi 2 + Fn+1

2 o o = Fn Fn+1 + Fn+1 , por hiptesis de induccin, para j = n,

= Fn+1 (Fn + Fn+1 ) = Fn+1 Fn+2 , por denicin de Fibonacci o Es decir, tenemos que se cumple (7).

18

1. Induccin Matemtica o a

Como puede notarse de la demostracin anterior, la induccin generalizada o o tiene como caso particular a la induccin matemtica usual. Puede demostrarse o a que el rec proco es tambin cierto. e En la siguiente propiedad haremos uso del principio de induccin generalo izada con toda su fuerza. Teorema 3. Sean , reales tales que = entonces n N, Fn = n n 5 (8) 1 1+ 5 , 2 = 1 1 5 2

Demostracin. Por supuesto, probaremos (8) usando induccin generalo o izada. Antes de comenzar notemos que las soluciones de la ecuacin o x2 x 1 = 0 son precisamente y . Por lo que se cumple 2 = + 1. Caso base: F0 = 0 y (0 0 )/ 5 = 0. Adems F1 = 1 y a 1 1 12 5 = 2 5 5 =1 Esto es, se cumple la ecuacin de (8) para n = 0 y n = 1. o Hiptesis de induccin: suponemos que o o Fj = n n , para 1 j n 5 2 = + 1, (9)

Paso inductivo: por demostrar que Fn+1 = n+1 n+1 5 (10)

3. Induccin matemtica generalizada o a

19

En efecto: Fn+1 = Fn + Fn1 = = = = = n n n1 n 1 + , por hip. de ind. para j = n, j = n 1, 5 5 n + n1 n n1 5 n1 (1 + ) n1 (1 + ) 5 n1 2 n1 2 5 n+1 n+1 5 esto es, se cumple (10).

Tarea 3. Se denen los nmeros cn como u cn = Demostrar que 1, 2n + cn1 1, si n = 1 si n > 1.

n N, cn = n2 . Tarea 4. Demostrar que los trminos de la sucesin de Fibonacci satisfacen e o las siguientes relaciones: (1)
n i=1 2 (2) F1 F2 + F2 F3 + F2n1 F2n = F2n

Fi = Fn+2 1

2 (3) Fn1 Fn+1 = Fn + (1)n para n 2.

Cap tulo 2

Matrices

Una matriz es un arreglo rectngular de nmeros encerrado por parntesis. Por a u e ejemplo 1 0 3 2 1 4 2 1 3 , 4 5 6 4 2 1 3 1 1 son matrices. El tamao o orden de una matriz est especicado por el nmero de las n a u y columnas que tiene. Si una matriz tiene n las y m columnas, entonces el orden de la matriz se dice que es n por m. Tal orden se denota como n m. Ejemplo 11. 2 1 3 4 2 1 tiene orden 2 3

La entrada o registro i, j de una matriz es el nmero situado en la la i y u columna j. Ejemplo 12. La matriz 2 1 3 4 2 1 tiene entrada 2, 3 a 1, entrada 2, 1 a 4, entrada 1, 3 a 3. 21

1 0 3 2 1 4 tiene orden 4 3 4 5 6 3 1 1

22

2. Matrices

o ms brevemente a o an ms breve u a

Para cuando se tiene una matriz a11 a21 A= . . .

an1 an2

A en abstracto, se acostumbra poner a12 a1n a22 a2n . . . . . . anm

A = (aij )1in,1im A = (aij ) . Ntese que los sub o ndices coinciden con las entradas: a ij est en la entrada a i, j.

1. Producto de matrices
Se pueden efectuar varias operaciones algebricas con las matrices. Examinemos a la ms complicada primero: la multiplicacin de matrices. a o Denicin 13. Sea A = (aij ) matriz de orden n m y B = (bij ) matriz de o orden m s. Entonces se dene la matriz producto AB como c11 c1s . . . AB = . . . cn1 cns
m

donde

cij =
k=1

aik bkj , para 1 i n, 1 j s

(11)

Notemos que para multiplicar dos matrices se requiere que el nmero de u columnas del primer factor coincida con el nmero de las del segundo factor. u Ejemplo 14. 4 1 2 5 2 3 2 3 0 1 1 = 4 6 2 2 3 5 3

8 + 9 4 2 + 0 + 6 4 + 3 + 10 10 + 3 6 16 + 18 + 4 4 + 0 6 8 + 6 10 20 + 6 + 6 =

3 8 9 13 38 10 4 32

1. Producto de matrices

23

Ejemplo 15. El producto 4 1 2 5 2 3 2 3 0 1 1 4 6 2 2 3 5 3 Ejemplo 16. Sean A= entonces 1 2 , 3 4 AB = mientras que BA = B= 0 1 2 5

no puede efectuarse, pues el numro de columnas del primer factor es 4 que e no coincide con el n mero de las del segundo, que es 2. u

4 9 8 17

3 4 . 13 16 Como puede notarse, en general, el orden de los factores altera el producto de matrices. Tarea 5. Sean A= 2 1 3 , 4 0 1 B= 4 1 2 , 5 1 3 2 1 6 , C= 0 3 2 4 2 D= 3 5 . 1 3

Calcular AB, CD, AC, DB, ADB, DBA, BDAC. Denicin 17. Sea In = (aij ) matriz n n o i = j. Es decir 1 0 0 0 1 0 In = . . . . . . . . .

tal que aii = 1, y aij = 0 si 0 0 . . .

0 0 0 1 Tal matriz In se llama matriz identidad n n.

La matriz identidad es una especie de elemento identidad, un uno; como puede notarse en la siguiente propiedad. Propiedad 7. Sea B = (bij ) matriz n m entonces

24

2. Matrices

(1) BIm = B (2) In B = B. Demostracin. Haremos uso de la notacin compacta para matrices. o o Pongamos In = (aij ). (1) Tenemos que el producto de matrices es una nueva matriz, por lo que podemos poner BIn = (cij ) donde cada entrada cij es
m

cij =
k=1

bik akj

= bij ajj , pues aij=0 si ai=j , = bij porque ajj = 1, (2) Tarea.

Tambin pudimos hacer uso de la notacin no compacta para matrices. Por e o ejemplo para demostrar el primer inciso de la propiedad anterior, pudimos hacer escrito:
BIm =

=B

b11 . . . bn1

b12 . . . bn2

b1m . . . bnm

Supngase la suma S, o

b11 + b12 0 + b1m 0 . . . bn1 + bn2 0 + bnm 0

b11 0 + b12 + b13 0 + + b1m 0 . . . bn1 0 + bn2 + bn3 0 + + bnm 0

b11 0 + + b1(m1) 0 + b1m . . . bn1 0 + + bn(m1) 0 + bnm

b11 b21 . . . bn1

b12 b22 . . . bn2

b1m b2m . . . bnm

1 0 . . . 0

0 1 . . . 0

0 0 . . . 1

1. Producto de matrices

25

S = a11 + a12 + . . . + a1m + a21 + a22 + . . . + a2m + ... + an1 + an2 + . . . + anm Usando sumatorias, tal suma S se puede escribir como
m

S=
j=1 m

a1j a2j
j=1

+ + ...
m

+
j=1

anj

y usando de nuevo sumatorias,


n m

S=
i=1 j=1

aij .

Ntese que tal frmula se obtuvo sumando primero por las. Si sumamos o o ahora por columnas:
n n a

S=
i=1 m

ai1 +
i=1 n

ai2 + +

aim
i=1

=
j=1 i=1

aij

Hemos demostrado: Teorema 4. Si aij R


n m m n

aij =
i=1 j=1 j=1 i=1

aij

Esta frmula la necesitamos para demostrar la asociatividad del producto o de matrices. Teorema 5 (asociativa del producto). Sean A = (a ij ) matriz n m, B = (bij ) matriz m s y C = (cij ) matriz s r. Entonces (AB)C = A(BC)

26

2. Matrices

Demostracin. Pongamos o AB = (dij ) que es una matriz n s. Entonces dij =


k=1 m

aik bkj .

(12)

Luego, ponemos (AB) C = (eij )


ns sr

que es una matriz n r, donde eij = =

dil clj
l=1 s m

aik bkl clj


l=1 k=1

(13)

usando la ecuacin (12). o Ahora pongamos BC = (ij ) con ij =


u=1 s

biu cuj

(14)

luego podemos poner A (BC) = (ij )


nm mr

con
m

ij =
v=1 m

aiv vj
s

=
v=1 m

aiv
u=1 s

bvu cuj , seg n (14), u aiv bvu cuj

=
v=1 u=1 s m

=
u=1 v=1 s m

aiv bvu cuj , por teorema 4 aik bkl clj , pues el nombre del ndice no importa
l=1 k=1

= = eij

2. Suma de matrices

27

seg n (13). Es decir u A(BC) = (eij ) = (ij ) = (AB)C.

2. Suma de matrices
Denicin 18 (suma y diferencia de matrices). Supongamos que A = (a ij ) o matriz n m, B matriz n m. Entonces a11 + b11 a12 + b12 aim + b1m . . . . . . A+B = . . . y an1 + bn1 an2 + bn2 anm + bnm a11 b11 a12 b12 . . . . AB = . . an1 bn1 an2 bn2 A= Sol. A+B = Ejemplo 20. Sean A= Hallar A + B. 1 1 , 2 4 B= 5 4 6 3 7 1 0 2 2 , 1 2 2 AB = 2 2 6 1 8 4 1 2 4 , 0 3 1 B= aim b1m . . .

anm bnm

Ejemplo 19. Sean

1 0 2 1 5 3

Sol. La suma no est denida pues A y B son matrices de diferente orden. a Denicin 21. Sea o Onm 0 . . .

0 0 la matriz con todas sus entradas cero. Tal matriz se llama matriz cero de orden n m Es comn dejar sobreentendido el orden de la matriz cero, y as eliminar sus u sub ndices. As 0, o 0 se usa para representar a la matriz cero.

0 . = . .

28

2. Matrices

Propiedad 8. Sean A = (ai j), B = (bij ) matrices de orden nm. Entonces (1) A + B = B + A (2) (A + B) + C = A + (B + C) (3) A + 0 = A Demostracin. o (1) Usamos la notacin abreviada para matrices: o A + B = (aij + bij ) = (bij + aij ), por conmutativa en R; = (bij ) + (aij ) =B+A (2) Tarea (3) Tarea

Propiedad 9. (1) Si A = (aij ) es matriz n m y B = (bij ), C = (cij ) son matrices m s entonces A(B + C) = AB + AC, (distributiva derecha) (2) Si A = (aij ), B = (bij ) son matrices n m y C = (cij ) es matriz m s, entonces (A + B)C = AB + AC, Demostracin. o (1) Tenemos que B + C = (bij + cij ) y entonces que A(B + C) = (dij ) donde
m

(distributiva izquierda)

(15) (16)

dij =
k=1 m

aik (bkj + ckj ), por (15) y def. de producto de matrices (aik bkj + aik ckj ) , distributiva en R,
k=1 m m

= =
k=1

aik bkj +
k=1

aik ckj

(17)

2. Suma de matrices

29

Por otro lado, AB = (ij ), donde ij =


u=1 m

AC = (ij )
m

aiu buj ,

ij =
u=1

aiu cuj

entonces AB + AC = (ij + ij )
m m

=
u=1 m

aiu buj +
u=1 m

aiu cuj aik ckj


k=1

=
k=1

aik bkj +

= (dij ), seg n (17) u = A(B + C), seg n(16). u (2) Tarea

Tarea 6. Sean A= 2 1 0 , 4 0 1 B= 4 1 2 , 5 1 3 2 1 6 , C= 0 3 2 4 2 D= 3 5 . 1 3 (A B)(C + D)

Calcular, en caso de ser posible, A + B, , B + C, , C D,

(A B)D,

Cap tulo 3

Sistemas de ecuaciones

Uno de los empleos ms populares de las matrices es la resolucin de sistemas a o de ecuaciones. Ejemplo 22. Resolver el sistema x2 3x3 = 5 2x1 + 3x2 x3 = 7 4x1 + 5x2 2x3 = 10 Sol. El sistema de ecuaciones es equivalente a 2x1 + 3x2 x3 = 7 x2 3x3 = 5 4x1 + 5x2 2x3 = 10

(18)

slo intercambiando la primera ecuacin con la segunda. Multiplicando la o o ahora primera ecuacin por 2: o 4x1 6x2 + 2x3 = 14 x2 3x3 = 5 4x1 + 5x2 2x3 = 10

Sumando a la tercera ecuacin la primera, lado a lado: o 4x1 6x2 + 2x3 = 14 x2 3x3 = 5 x2 = 4 multiplicando la primera ecuacin por 1/2: o 2x1 + 3x2 x3 = 7 x2 3x3 = 5 x2 = 4 31

32

3. Sistemas de ecuaciones

sumando la segunda ecuacin a la tercera, lado a lado, o 2x1 + 3x2 x3 = 7 x2 3x3 = 5 3x3 = 9

despejando x3 de la ultima ecuacin obtenemos que x 3 = 3. Enseguida o hacemos sustitucin regresiva en las ecuaciones anteriores; sustituyendo en o la segunda ecuacin, queda x2 9 = 5, es decir, x2 = 4. Sustituimos ahora o en la primer ecuacin: 2x1 + 12 3 = 7, es decir x1 = 1. Por lo tanto, las o soluciones son: x1 = 1, x2 = 4, x3 = 3 . Como puede notarse en el ejemplo, el uso de los s mbolos x 1 , x2 , x3 es un tanto redundante. Por lo que las operaciones hechas se pueden abreviar de la forma siguiente. Primero distinguimos los coecientes del sistema original de ecuaciones (18), junto con los trminos constantes que aparecen del lado e derecho de las ecuaciones: 0 1 3 5 2 3 1 7 4 5 2 10 matriz. Primero intercambi 3 1 7 1 3 5 5 2 10

enseguida aplicamos ciertas operaciones sobre la amos la primera con la segunda la: 0 1 3 5 2 2 3 1 7 1F 2F 0 4 5 2 10 4

ahora sumamos a la tercer la la segunda, de nuevo 2 3 2 3 1 7 +3F 0 1 3 5 2F 0 1 0 1 0 4 0 0

luego multiplicamos la primera la por 2 y sumamos a la tercera, registro a registro: 2 3 1 7 2 3 1 7 0 1 3 5 1F (2)+3F 0 1 3 5 4 5 2 10 0 1 0 4 registro a registro: 1 7 3 5 3 9

y es entonces que escribimos esta matriz como un sistema de ecuaciones: 7 2x1 + 3x2 x3 = x2 3x3 = 5 3x3 = 9 sistema que es muy fcil de resolver. a

1. El mtodo de Gauss e

33

El mtodo expuesto se llama de Gauss. e

1. El mtodo de Gauss e
Denicin 23. En un sistema de n ecuaciones con m incgnitas: o o a11 x1 a21 x1 + a12 x2 + a22 x2 + + + a1m xm + a2m xm = b1 = b2 . . .

an1 x1 + an2 x2 + la matriz a11 . A= . .

+ anm xm = bn a1m . . .

se llama matriz aumentada.

se llama matriz de coecientes. Mientras que la matriz a11 a1m b1 . . . . . A|b = . . . . an1 anm bn

an1

anm

En el mtodo de Gauss se permiten solamente las siguientes operaciones e entre matrices (llamadas operaciones elementales por las): Multiplicar una la por un nmero no cero u Intercambiar dos las

Sumar dos las

Ejemplo 24. Hallar la solucin del sistema o x1 2x2 + x3 = 7 2x1 5x2 + 2x3 = 6 3x1 + 2x2 x3 = 1 Sol. 1F (2)+2F 1 2 1 7 2 5 2 6 2F (3)+3F 3 2 1 1 1 2 1 7 2F (1) 0 1 0 8 0 8 4 20 1 2 1 7 0 1 0 8 0 8 4 20 1 2 1 7 2F (8)+3F 0 1 0 8 0 0 4 84

34

3. Sistemas de ecuaciones

es decir, el sistema original es equivalente a x1 2x2 + x3 = 7 x2 = 8 4x3 = 84 y por sustitucin regresiva se obtiene que o x3 = 21, x2 = 8, x1 = 2

Los nmeros con los que se hacen ceros debajo de ellos se llaman pivotes. u Una matriz n n de la forma 0 0 0 . . . . . . . . . . . . 0 0 0 se llama matriz triangular superior. Mientras que una de la forma . . . se llama triangular inferior. Como puede notarse el objetivo del mtodo de Gauss es transformar la e matriz aumentada de un sistema de ecuaciones a una triangular superior (si el sistema tiene igual nmero de ecuaciones que de incgnitas). u o 0 . . . 0 0 0 0 . . . . . . 0

Ejemplo 25. Resolver 2x1 4x2 x1 3x2 + x4 x1 x3 + 2x4 3x1 4x2 + 3x3 x4 = 10 = 4 = 4 = 11

1. El mtodo de Gauss e

35

Es decir, el sistema original es equivalente a x1 2x2 x2 x3

La matriz aumentada del sistema es 4 0 0 10 1 2 0 0 5 3 0 1 4 1F 1/2 1 3 0 1 4 1 4 4 0 1 2 0 1 2 4 3 1 11 3 4 3 1 11 1F (1)+2F 1 2 0 0 5 1 2 0 0 5 1F (1)+3F 1F (3)+4F 0 1 1 2F (1) 0 0 1 1 0 1 1 0 0 2 1 2 2 1 2 9 9 4 4 0 2 3 1 0 2 3 1 1 2 0 0 5 1 2 0 0 5 2F (2)+3F 3F (2)+4F 0 1 0 1 1 3F (1) 0 1 0 1 1 0 0 0 1 4 11 0 1 4 11 0 0 3 1 6 0 0 3 1 6 1 2 0 0 5 1 2 0 0 5 3(3)+4F 0 1 0 1 1 4(1/13) 0 1 0 1 1 0 0 0 1 4 11 0 1 4 11 0 0 0 13 39 0 0 0 1 3 Sol. 2 1 1 3 = 5 x4 = 1 4x4 = 11 x4 = 3

Tarea 7. Resolver los sistemas siguientes usando el mtodo de Gauss con e sustitucin regresiva. o (1) 2x y = 8 6x 5y = 32 4x1 3x2 = 10 3x1 x2 = 10 2x + y 3z = 0 6x + 3y 8z = 0 2x y + 5z = 4

que puede ser resulto fcilmente por sustiticin regresiva: x 4 = 3; sustia o tuyendo en la tercera ecuacin x3 12 = 11, i.e., x3 = 1; de la segunda o ecuacin obtenemos que x2 = 2 y de la primera que x1 = 1. La solucin o o es x1 = 1, x2 = 2, x3 = 1, x4 = 3

(2)

(3)

36

3. Sistemas de ecuaciones

(4) x1 + 2x2 x3 = 3 3x1 + 7x2 + 2x3 = 1 4x1 2x2 + x3 = 2 x1 2x1 (6) 3x1 6x2 x1 2x2 2x1 3x2 x1 x1 2x1 3x1 3x2 2x2 8x2 9x2 2x2 x3 4x2 + x3 7x2 2x3 3x2 + x4 2x4 + x4 4x4 = 6 = 9 = 10 = 16 = 5 = 18 = 8 = 4 = 2 = 1 = 3 = 7

(5)

x3 2x4 + 2x3 + x3 x4 + 3x4 + x3 + 2x4 + 2x3 + 4x4 x3 + 4x3

(7)

No todos los sistemas de ecuaciones tienen solucin: o Ejemplo 26. Resolver: 2x2 2x1 x1 x1 + x2

+ + +

2x3 4x3 2x3 3x3

4x4 2x4 x4 + 3x4

= = = =

1 0 1 4

Sol. Por el mtodo de Gauss: e


F 1F 2

1F 1/2

2F (1/2)

2F (1)+F 4

1 0 0 0

0 1 0 1

2 1 0 1

1 2 0 2

0 1/2 1 4

1 0 0 0

0 1 0 0

2 1 0 0

1 2 0 0

0 1/2 1 7/2

1F (1)+F 3 1F +4F

1 0 1 1

0 2 0 1

2 2 2 3

1 4 1 3

0 1 1 4

1 0 0 0

0 2 0 1

2 2 0 1

1 4 0 2

0 1 1 4

0 2 1 1

2 0 0 1

2 4 2 3

4 2 1 3

1 0 1 4

2 0 1 1

0 2 0 1

4 2 2 3

2 4 1 3

0 1 1 4

1. El mtodo de Gauss e

37

la aparicin de sta ultima matriz indica que el sistema de ecuaciones origo e inal es equivalente a x1 + 2x3 x4 x2 x3 + 2x4 0 0 = 0 = 4 = 1 = 7/2

siendo que las dos ultimas ecuaciones indican contradicciones. Se concluye que el sistema de ecuaciones no tiene solucin. o

En la parte nal del mtodo de Gauss se pueden hacer sustituciones ree gresivas. Tales sustituciones se pueden abreviar en las matrices mismas: ste e mtodo se conoce como de Gauss-Jordan. e Ejemplo 27. Hallar la solucin de o x1 2x2 + x3 = 7 2x1 5x2 + 2x3 = 6 3x1 + 2x2 x3 = 1 Sol. 1F (1)+2F 1 2 1 7 1 2 1 7 2F (1) 1F (3)+3F 2 5 0 1 2 6 0 8 3 2 1 1 0 8 4 20 2F 2+1F 1 2 1 7 1 0 1 23 2F (8)+3F 3F (1/4) 0 0 1 1 0 8 0 8 0 8 4 20 0 0 4 84 1 0 1 23 1 0 0 2 3F (1)+1F 0 1 0 0 1 0 8 8 0 0 1 21 0 0 1 21 x1 = 2, x2 = 8, x3 = 21.

Por lo tanto las soluciones son

Tarea 8. Resolver (1) 3x1 2x2 = 8 4x1 + 5x2 = 3

38

3. Sistemas de ecuaciones

(2) x1 4x2 + x3 = 8 3x1 12x2 + 5x3 = 26 2x1 9x2 x3 = 14 2x1 4x2 x1 3x2 + x4 x1 x3 + 2x4 3x1 4x2 + 3x3 x4 x1 2x1 x1 4x1 (5) 2x1 x1 3x1 2x1 Obsrvese que: e el mtodo de Gauss en un sistema con tantas ecuaciones como e incgnitas termina cuando se obtiene una matriz tringular superior; o el mtodo de Gauss-Jordan en un sistema de n ecuaciones con n e incgnitas termina cuando se obtiene una matriz de la forma o 1 0 0 0 b1 0 1 0 0 b2 . . . . . . . . . . . . . . . 0 0 0 1 bn + 2x2 + 5x2 2x2 + 10x2 3x3 6x3 + x3 x4 9x3 + x4 = 10 = 4 = 4 = 11 = 8 = 17 = 8 = 33 = 0 = 7 = 19 = 12

(3)

(4)

+ 3x2 x4 + x2 + x3 2x4 x2 + 2x3 + x4 4x2 + 3x3

As como hay sistemas de ecuaciones sin soluciones, los hay con una innidad: Ejemplo 28. Resolver el sistema de ecuaciones x1 x 2 = 0 2x1 2x2 = 0 Sol. La matriz aumentada del sistema es 1 1 0 2 2 0 1 1 0 0 0 0

2. Sistemas de ecuaciones lineales generales

39

es decir el sistema es equivalente a x1 x 2 = 0 que tiene como soluciones a x1 = x2 = 1, x1 = x2 = por lo que se tienen una innidad de soluciones. 2, x1 = x2 = 7, etc;

Como podemos observar, los sistemas de ecuaciones a veces tienen solucin o unica, otras veces no tienen solucin; mientras que a veces tienen una innidad o de soluciones. Podemos entonces clasicar los sistemas de ecuaciones lineales como si indica en la gura 1.
sistema de ecuaciones lineal

hay solucion? no sistema de ecuaciones inconsistente si sistema de ecuaciones consistente

solucion unica? no sistema de ecuaciones con infinidad de soluciones si sistema de ecuaciones con solucion unica

Figura 1. Clasicacin de los sistemas de ecuaciones lineales o

Que resulten una innidad de soluciones es frecuente cuando se tienen sistemas de ecuaciones rectngulares, con ms incgnitas que ecuaciones. En a a o todos lo ejemplos anteriores se han usado sistema de ecuaciones lineales cuadrados, esto es, sistemas con igual nmero de incgnitas que de ecuaciones. u o

2. Sistemas de ecuaciones lineales generales


En general, cuando se tiene un sistema de ecuaciones rectangular, el procediemiento de solucin que se puede emplear es el de Gauss; similarmente o a los sistemas cuadrados. Lo unico que hay que cambiar el la condicin de o nalizacin de tal algoritmo. El mtodo de Gauss naliza cuando se obtiene o e una matriz escalonada por las. Denicin 29. Una matriz se dice que est en forma escalonada por las o a si

40

3. Sistemas de ecuaciones

(1) Todas las las que tienen slo ceros aparecen debajo de las las con o entradas diferente de ceros. (2) La primera entrada de la primera la es no cero. Despus de la e primera la: una primera entrada no cero de cualquier la aparece en una columna a la derecha de la primera entrada no cero de la la anterior.

Esto signica que una matriz est en forma escalonada si tiene la forma a 0 0 0 0 0 0 0 . . . . . . 0 0

0 0 . . . 0

0 0 0 0 . . . . . . . . . 0 0 0

Ejemplo 30. Determinar si alguna de las matrices siguientes est en forma a escalonada por las. 1 3 2 0 0 0 , 0 0 1 2 4 0 1 3 2 , 0 0 0 1 0 0 0 3 0 0 0 2 1 0 0 5 3 , 1 0 1 1 1 0 0 0 2 0 0 0 0 1 0 0 0 4

Sol. La primera matriz no, pues la la de ceros deber estar en la tercer a la. La segunda matriz tampoco, porque la entrada 2, 1 deber de ser cero. a La tercer y cuarta matriz si estn en forma escalonada por las. a Ejemplo 31. Reducir la matriz 2 4 2 2 2 4 3 4 4 8 3 2 0 0 1 2 mediante operaciones elementales por las a una matriz escalonada.

2. Sistemas de ecuaciones lineales generales

41

Sol. 2 4 2 2 2 4 2 2 2 4 3 4 0 0 1 1 4 8 3 2 0 0 1 2 0 0 1 2 0 0 1 2 2 4 2 2 2 4 2 2 0 0 1 1 0 0 1 1 0 0 0 1 0 0 0 1 0 0 0 1 0 0 0 0

Tarea 9. Reducir las siguientes matrices (operaciones elementales por las) a una forma escalonada. 0 0 3 2 2 4 2 2 1 4 1 3 2 , 4 8 3 , 0 0 1 2 , 1 3 2 4 1 3 0 3 1 6 0 2 1 3 0 0 1 2 1 4 1 1 2 0 0 0 0 1 1 3 1 1 3 3 , 2 4 1 3 2 1 1 5 5 9

Ejemplo 32. Hallar todas las soluciones de un sistema lineal de ecuaciones cuya matriz aumentada se puede reducir por las a 1 3 0 5 0 4 0 0 1 2 0 7 0 0 0 0 1 1 0 0 0 0 0 0 Sol. El sistema original es equivalente a x1 3x2 5x4 x3 + 2x4 x5 = 4 = 7 = 1

depejando x1 = 3x2 5x4 + 4 x2 = 2x4 7 x5 = 1

Obsrvese que no se ha despejado x2 ni x1 . Esto signica que pueden e tomar cualquier valor, es decir, estn libres, se les puede asignar cualquier a

42

3. Sistemas de ecuaciones

valor. Simbolizamos semejante hecho por las ecuaciones x 4 = s, x2 = r. Por tanto las soluciones estn dadas por a x1 = 3r 5s + 4 x2 = r x3 = 2s 7 x4 = s x5 = 1 Podemos escribir esta ecuacin utilizando vectores: o x1 3r 5s + 4 x2 r x3 = 2s 7 , r, s R x4 s x5 1 Ejemplo 33. Encontrar todas las soluciones de x1 2x2 + x3 x4 = 4 2x1 3x2 + 2x3 3x4 = 1 3x1 5x2 + 3x3 4x4 = 3 Sol. Por el mtodo de Gauss: e 1F (2)+2F 4 4 1 2 1 1 1 2 1 1 2 3 2 3 1 2F (3)+3F 0 1 0 1 9 3 3 5 3 4 0 1 0 1 9 0 2 1 1 4 2F (1)+3F 0 1 0 1 9 0 0 0 0 0

siendo sta ultima matriz una matriz escalonada, lo que indica que no hay e ms que hacer con el mtodo de Gauss. Escribimos la ultima matriz como a e el sistema de ecuaciones que representa. x1 2x2 + x3 x4 = 4 x2 x4 = 9 Despejamos ahora las variables con sub ndice menor en trminos de la varie ables con sub ndice mayor: x1 = 2x2 x3 + x4 + 4 x2 = x 4 + 9

2. Sistemas de ecuaciones lineales generales

43

tal depeje es imposible de hacer para x 3 y x4 . Por lo que concluimos que esn libres: x4 = s, x3 = r y as a , x4 = s x3 = r x2 = s 9 o en forma vectorial: x1 = 2(s 9) r + s + 4 = 3s r 14 x1 3s r 14 x2 s 9 = , x3 r s x4

r, s R.

Tarea 10. Describir todas las soluciones x1 x1 2x2 = 3 3x1 3x1 x2 = 14 3x1

de los sistemas 3x2 + x3 = 2 8x2 + 2x3 = 5 7x2 + x3 = 1

4 x1 + 4x2 2x3 = 2x1 + 7x2 x3 = 2 2x1 + 9x2 7x3 = 1 2x3 + x4 = 6 x1 2x1 x2 + x3 3x4 = 0 9x1 3x2 x3 7x4 = 0 3x1 + 4x2 5x3 + x4 + 5x5 2x6 = 24 4x1 + 3x2 23x3 + x4 x6 = 9 5x1 + 6x2 13x3 + 2x4 + 5x5 8x6 = 21

Cap tulo 4

Determinantes

1. Semntica geomtrica a e
1.1. Vectores. Las matrices de orden 1 n se llaman vectores la: (a1 , a2 , . . . , an ), vector la. Mientras que las matrices de orden n 1 se llaman vectores columna: a1 a2 . , vector columna. . . an s : (a1 , . . . , an ) + (b1 , . . . , bn ) = (a1 + b1 , . . . , an + bn ) Por supuesto, vectores de diferente tamao no pueden sumarse ni restarse entre n s . El conjunto de todos los vectores de tamao n con entradas en R se denota: n Rn = {(a1 , . . . , an ) | a1 , . . . , an R} Las anteriores no son las unicas operaciones que se pueden hacer entre vec tores. Una operacin muy comn en vectores es la multiplicacin por escalares. o u o Denicin 34. Sea c R, v = (a1 , . . . , an ) Rn . Se dene el producto por o el escalar c de v como cv = (ca1 , . . . , can ) 45 (a1 , . . . , an ) (b1 , . . . , bn ) = (a1 b1 , . . . , an bn ).

Luego, como matrices que son, los vectores pueden restarse o sumarse entre

46

4. Determinantes

Cuando se habla de vectores, uno se reere a los nmeros como escalares. u Propiedad 10. Sean c, d R, v, w Rn . Entonces (1) c(v + w) = cv + cw (2) (c + d)v = cd + dv (3) 1v = v Demostracin. o (1) Pongamos v = (a1 , . . . , an ) y w = (b1 , . . . , bn ). Entonces c(v + w) = c(a1 + b1 , . . . , an + bn ), por denicin de suma enRn o = (c(a1 + b1 ), . . . , c(an + bn )), seg n def. de prod. por escalar, u = (ca1 + cb1 , . . . , can + cbn ), distributiva en R, = (ca1 , . . . , can ) + (cb1 , . . . , cbn ), por def. de suma en Rn , = c(a1 , . . . , an ) + c(b1 , . . . , bn ), por def. de producto por escalar, = cv + cw (2) Tarea (3) Tarea

A los vectores en R2 se acostumbra representarlos como echas en el plano: si (x1 , x2 ) R2 entonces se pone una echa con inicio el vector origen (0, 0) y punta en el punto que resulta de desplazarse x 1 unidades en el eje x y x2 unidades en el eje y. y

y1
   

Q (x1 , y1 ) 

 s

(0, 0)

x1

Por ejemplo las representaciones de (1, 1), (1, 2), (3, 1) son

1. Semntica geomtrica a e

47

2 1 (1, 1) 

f 1 1 f 2 f 1 f f  f (1, 2) 2 f f x f

3 (1, 3)

As la suma de vectores en R2 se representa como la diagonal principal de un paralelogramo:

y (a1 , a2 ) + (b1 , b2 )
} # (b , b )  1 2 (a1 , a2 ) }

Para el producto por escalares c(a1 , a2 ) hay varias posibilidades, dependiendo del escalar c. Si c > 1 entonces el vector c(a 1 , a2 ) es ms largo que a (a1 , a2 ) pero tiene la misma direccin: o

48

4. Determinantes

y


c(a1 , a2 )

c>1

 1 , a2 ) (a

mientras que para 0 < c < 1 el vector c(a 1 , a2 ) se reduce:

0<c<1
 1 , a2 ) (a

 c(a1 , a2 )

y nalmente si c < 0 entonces el vector original cambia de direccin: o

1. Semntica geomtrica a e

49

c<0
 1 , a2 ) (a

c(a1 , a2 ) Tarea 11. Dibujar los siguientes vectores en el plano: (2, 1), (1, 3), (3, 1), (2, 4), (4, 2). Tarea 12. Encontrar todos los valores posibles de las x i s tales que (2) 4(x1 , x2 ) + 2(x1 , 3) = (6, 18) (3) (x1 , x2 ) (4) x1 x2 x3 x4 1 3 =2 0 1 3 1 (1) 2(x1 , x2 ) (4, 7) = (2, 11)

1 1 1 0

1.2. Angulo entre vectores. Cun largo es un vector se mide con la norma. a Denicin 35. Si (a1 , a2 , . . . , an ) Rn , se dene su norma como o ||(a1 , a2 , . . . , an )|| = a2 + a 2 + + a 2 n 2 1 Ejemplo 36. Las normas de los vectores v 1 = (1, 1), v2 = (1, 2, 3), v3 = (0, 1, 2, 2, 2) son, respectivamente, ||v1 || = 12 + (1)2 = 2

||v2 || =

1+4+9 = 14

12 + 22 + 32

50

4. Determinantes

||v3 || =

02 + 12 + 2 + 2 + 2 2 = 9=3

Tarea 13. Para los vectores dados v, w calcule ||v||, ||w||, ||v +w||, ||v w||, ||2v||, y ||3w||. (1) v = (1, 2, 1), w = (0, 1, 4) (2) v = (0, 1, 2, 2, 1), w = (3, 1, 2, 1, 2) (3) Es cierto que ||v + w|| = ||v| + ||w| ? (4) Es cierto que ||v w|| = ||v|| ||w|| ? Tarea 14. Si v Rn y c R demostrar que ||cv|| = |c| ||v|| Nos disponemos a encontrar una frmula algebrica para el angulo entre o a dos vectores:

(b1 , b2 )
  0 

(a1 , a2 )

  

x
Figura 1. Angulo entre vectores

Recordemos que en un tringulo rectngulo de hipotenusa h: a a

1. Semntica geomtrica a e

51

$$ $$$ s (2, 1, 3) s d e ed e e e e s (1, 1, 1)

$ $$$

$s $$ (1, 2, 0)

el seno y el coseno del angulo son sin() = cos() = lado opuesto hipotenusa

lado adyacente . hipotenusa

Utilizaremos tales ecuaciones para el tringulo de la Figura 1. El problema a es que tal tringulo, no es, en general un tringulo rectngulo: a a a y

(b1 , b2 )
  0 

(a1 , a2 )

  

x Sin embargo si prolongamos el lado correspondiente al vector (b 1 , b2 ), podemos obtener un tringulo rectngulo: por lo que a a cos() = y ||(b1 , b2 )|| + u ||(a1 , a2 )|| v ||(a1 , a2 )|| (19) (20)

sin() =

52

4. Determinantes

y u (b1 , b2 ) 
  0     

v
B

  

(a1 , a2 )

x
Figura 2. Extendiendo el lado de (b1 , b2 )

Notemos ahora que en el trngulo rectngulo de la Figura 2 existe un tringulo a a a rectngulo subyacente: a y u (b1 , b2 ) 
  0     

v
B

  

(a1 , a2 )

x donde ahora la hipotenusa es (a1 , a2 ) (b1 , b2 ). Utilizando el teorema de Pitgoras: a u2 + v 2 = ||(a1 , a2 ) (b1 , b2 ))||2 pero, despejando de (19) y de (20), u = ||(a1 , a2 || cos() ||(b1 , b2 )||, v = ||(a1 , a2 )|| sin() (21)

ecuaciones que sustituimos en (21) para obtener (||(a1 , a2 )|| cos() ||(b1 , b2 )||)2 + ||(a1 , a2 ||2 sin2 () = ||(a1 b1 , a2 b2 )||2

1. Semntica geomtrica a e

53

desarrollando el binomio al cuadrado del lado izquierdo: ||(a1 , a2 ||2 (cos2 () + sin2 ()) 2||(a1 , a2 )|| ||(b1 , b2 )|| = ||(a1 b1 , a2 b2 )||2
1

ahora desarrollando las normas al cuadrado: a2 + a2 + b2 + b2 2||(a1 , a2 || ||(b1 , b2 )|| cos() = (a1 b1 )2 + (a2 b2 )2 1 2 1 2 cancelando valores comunes queda = a2 2a1 b1 + b2 + a2 2a2 b2 + b2 1 1 2

Para escribir de una forma ms compacta sta ecuacin hacemos uso de la a e o siguiente denicin: o Denicin 37. Si v = (a1 , . . . , an ), w = (b1 , . . . , bn ) Rn entonces se dene o el producto punto (interno, fundamental, escalar) como v w = a 1 b1 + . . . + a n bn Luego, la ecuacin (22) queda como o ||(a1 , a2 )|| ||(b1 , b2 )|| cos() = (a1 , a2 ) (b1 , b2 ) Hemos demostrado, Teorema 6 (Angulo entre vectores). Si es el angulo entre dos vectores 2 entonces v, w R v w = ||v|| ||w|| cos() Cuando v = 0 y w = 0 tal frmula se puede usar como o = arc cos Teorema 7. Si v, w R2 entonces vw ||v|| ||w|| (23)

||(a1 , a2 )|| ||(b1 , b2 )|| cos() = a1 b1 + a2 b2

(22)

|v w| ||v|| ||w||

Demostracin. Por la frmula para el angulo entre vectores, tenemos que o o |v w| = | ||v|| ||w|| cos() | = ||v|| ||w|| | cos()|, pues ||v||, ||w|| son no negativos,

||v|| ||w||, usando que || cos()|| 1.

Ms propiedades del producto punto son las siguientes. a

54

4. Determinantes

Propiedad 11. Si v, w, u Rn entonces (2) v (w + u) = v w + v u (3) v w = w v Demostracin. o (1) Pongamos v = (a1 , . . . , an ). Entonces, por denicin del producto o punto v v = v 1 v1 + + v n vn
2 2 = v1 + + v n

(1) ||v||2 = v v

= ||v||2

seg n la denicin de norma. u o (2) Tarea (3) Tarea

Ntese como las propiedades de los vectores se pueden demostrar haciendo o expl citas las entradas de stos. No siempre es as como se muestra en el e , siguiente teorema. Teorema 8 (Desigualdad del tringulo). Si v, w R 2 , a ||v + w|| ||v|| + ||w|| Demostracin. o ||v + w||2 = (v + w) (v + w) =vv+vw+wv+ww = ||v||2 + 2v w + ||w||2 ||v||2 + 2|v w| + ||w||2 , pues x R, |x| x, ||v||2 + 2||v|| ||w|| + ||w||2 = (||v|| + ||w||)2

Hemos demostrado que ||v + w||2 (||v|| + ||w||)2 sacando ra ces cuadradas a ambos lados obtenemos la armacin del teoo rema.

1. Semntica geomtrica a e

55

1.3. Determinantes 2 2. Asi como calculamos una frmula para el angulo o entre vectores, desarrollaremos ahora una frmula para el area del paralogramo o determinado por dos vectores v = (a1 , a2 ), w = (b1 , b2 ) R2 : y
B # B

# v w

x
Figura 3. Paralelogramo determinado por v, w

Si enderezamos el paralelogramo de la Figura 1.3 obtenemos w




h v

E 

donde h es la altura del paralelogramo. Se sabe que el area A de tal paralel ogramo es base por altura: A = ||v|| h. Pero sin() = h/||w|| o despejando h = ||w|| sin(). Por lo que A = ||v|| ||w|| sin() = ||v|| ||w|| elevando al cuadrado, A2 = ||v||2 ||w||2 (1 cos2 ()) = ||v||2 ||w||2 ||v||2 ||w||2 cos2 () (1 cos2 ()), pues cos2 () + sin2 () = 1,

= (a2 + a2 )(b2 + b2 ) (a1 b1 + a2 b2 )2 1 2 1 2 = (a1 b2 a2 b1 )2

= ||v||2 ||w||2 (v w)2 , por la frmula del angulo entre vectores, o = a2 b2 + a2 b2 + a2 b2 + a2 b2 a2 b2 a2 b2 2a1 b1 a2 b2 2 2 2 1 1 1 1 2 2 1 1 1

56

4. Determinantes

Tenemos entonces A2 = (a1 b2 a2 b1 )2 y sacando ra cuadrada z A = |a1 b2 a2 b1 | (24)

Podemos escribir de manera ms compacta tal frmula usando la siguiente a o denicin. o Denicin 38. Sea B la matriz o B= a1 a2 b1 b2

entonces se dene el determinante de B como det B = |B| = a1 b2 a2 b1 Por lo que podemos escribir la frmula (24) como o A = | det B| Hemos demostrado:

Teorema 9. El area del paralelogramo determinando por los vectores v y w de la Figura 1.3 es al valor absoluto del determinante de la matriz formada con los vectores v y w como las. Ejemplo 39. Calcular el determinante de la matriz 2 3 1 4 Sol. 2 3 =83 =5 1 4 Ejemplo 40. Hallar el area del paralelogramo en R 2 con vrtices (1, 1), e (2, 3), (2, 1) y (3, 3). Sol. El area del paralelogramo requerido

1. Semntica geomtrica a e

57

(2, 3) r

r (3, 3)

(1, 1)


(2, 1) r

es el area del paralelogramo determinado por las restas (2, 3) (1, 1) = (1, 2) (2, 1) (1, 1) = (1, 0)

(1, 2)
! E

(1, 0) por lo que el area del paralelogramo pedido es el valor absoluto del deter minante 1 2 = 2 1 0 i.e., el area es 2. Tarea 15. (1) Calcular 1 3 , 5 0 0 3 , 5 0 1 0 , 0 7 21 4 , 10 7 cos() sin() sin() cos()

(2) Demostrar que a1 a2 = 0, a1 a2 b b a1 a2 = 1 2 a1 a2 b1 b2

58

4. Determinantes

1.4. Determinantes 3 3. Para localizar un punto en el espacio se usan tres rectas espaciales (en 3D) perpendiculares entre s Pare dibujar tales . usaremos perspectiva simple. Esto signica que los dibujos de nuestros objetos espaciales sern como sombras de objetos en tres dimensiones con una fuente a de luz cercana tanto al objeto a dibujar como a la sombra de tal:

&

&

&

&

Cada uno de ess rectas mutuamente ortogonales tiene nombre: x, y, z respeca tivamente. Un objeto en 3D tiene tres caracter sticas (al menos): largo, ancho y altura. As podemos decir que el eje x es para medir el largo, el y para el ancho , y el z para la altura.

T z

Por ejemplo el punto P (1, 1, 1) se dibuja como

1. Semntica geomtrica a e

59

T z

1
s P

O 1
x

mientras que el punto R(2, 1, 2) se dibuja


s

2
5 2 E

1
x

y el punto S(1, 2, 3)

T 5

1
x

O
s

2 E

2 As como los puntos del plano tienen asociados vectores posicin, los vec o tores en el espacio euclidiano tambin. Como antes, estos vectores posicin son e o vectores que tienen punto inicial en el origen O del espacio euclidiano y punto

60

4. Determinantes

nal el punto en cuestin. En los ejemplos anteriores, los vectores posicin o o relacionados se dibujan como
T z

1
s P  1 O 1 x # sR E

5 2 1 1 1 E O y 1 x

T 5

1
x

d s d

2 E

2 Tarea 16. Dibujar los siguientes puntos del espacio coordenado, as como sus vectores posicin: (1, 1, 2), (1, 1, 5), (2, 2, 2). o

1. Semntica geomtrica a e

61

Si v, w R3 entonces tambin vale la frmula del angulo entre vectores e o v w = ||v|| ||w|| cos() donde es el angulo entre los vectores v, w. Entonces el paralelogramo en R 3 : z
$$ w $ $ $$$ $$$  $ $X U   

&

&

&

&

&

&

&

 $$ $  $$

U  

$ $$$

$ $ $X

x A = ||v|| h

tiene area

donde h es la altura de tal paralelogramo. Pero como sin() = h/||w||, queda A = ||v|| ||w|| sin() = ||v|| ||w|| lo que implica A2 = ||v||2 ||w||2 ||v||2 ||w||2 cos2 () = ||v 2 ||||w||2 (v w)2 por lo que si ponemos coordenadas: v = (a 1 , a2 , a3 ), w = (b1 , b2 , b3 ), resulta A2 = (a2 + a2 + a2 )(b2 + b2 + b2 ) (a1 b1 + a2 b2 + a3 b3 )2 1 2 3 1 2 3 = a 2 b2 + a 2 b2 + a 2 b2 1 1 1 2 3 3 +a2 b2 + a2 b2 + a2 b2 2 1 2 2 2 3 +a2 b2 + a2 b2 + a2 b2 3 2 3 3 3 1 = (a1 b2 a2 b1 )2 + (a1 b3 a3 b1 )2 + (a2 b3 a3 b2 )2 De nuevo, para simplicar la notacin se dene: o a2 b2 a2 b2 a2 b2 2a1 b1 a2 b2 2a1 b1 a3 b3 2a2 b2 a3 b3 1 1 2 2 3 3 1 cos2 ()

62

4. Determinantes

Denicin 41 (Producto cruz). Si v = (a 1 , a2 , a3 ), w = (b1 , b2 , b3 ) R3 o entonces el producto cruz de v con w es vw = a a a a a2 a3 k 1 3 + 1 2 b1 b2 b1 b3 b2 b3

donde = (1, 0, 0), = (0, 1, 0), = (0, 0, 1). k Por tanto, sacando ra cuadradas: ces A2 = ||v w||2 . A = ||v w||. Hemos demostrado Teorema 10. El area A del paralelogramo determinado por los vectores v, w R3 es A = ||v w|| Ejemplo 42. Hallar el area del paralelogramo de R 3 determinado por v = (3, 1, 0), Sol. Tal area A es la norma de vw = 1 0 3 0 3 1 + k 3 2 1 2 1 3 = 2 6 + 8k = (2, 6, 8) w = (1, 3, 2)

por lo que A = ||(2, 6, 8|| = 4 + 36 + 64 = 104 = 4 26 = 2 26 Ejemplo 43. Hallar el area del tringulo en R 3 con vrtices (1, 2, 0), a e (2, 1, 3), (1, 1, 1) Sol. El dibujo correspondiente es

1. Semntica geomtrica a e

63

$$ $$$ s (2, 1, 3) s d e ed e e e e s (1, 1, 1)

$ $$$

$s $$ (1, 2, 0)

y trasladamos el punto (2, 1, 3) al origen para que el area pedida sea la mitad del area del paralelogramo determinado por las restas por lo que el area pedida es (1, 2, 0) (2, 1, 3) = (3, 1, 3), (1, 1, 1) (2, 1, 3) = (1, 0, 4) 98

||(3, 1, 3) (1, 0, 4)|| = ||(4, 9, 1)|| = Tarea 17. En los incisos siguientes hallar a b. (1) a = 2 + 3 b = + 2 k, (2) a = 5 + + 3 b = 2 + 3 k, k (3) a = + 2 + 4 b = 2 + 3 k, k b = 4 6 + (4) a = 2 + 3 k, k

Tarea 18. En cada inciso hallar el area del paralelogramo con vrtices en e el origen y con los vectores dados como aristas. (1) + 4 y 2 + 3 i (2) + 3 5 y 2 + 3 + i k k (3) hati 4 + y 2 + 3 k

Tarea 19. Hallar el area de (2) El tringulo con vrtices (2, 1, 3), (3, 0, 4), (1, 0, 5) a e (3) El paralelogramo con vrtices (1, 3), (2, 6), (1, 11), (4, 8) e (4) El paralelogramo con vrtices (1, 0, 1), (3, 1, 4), (0, 2, 9), (2, 1, 6) e (5) El paralelogramo acotado por las rectas x 2y = 3, x 2y = 10, 2x + 3y = 1, 2x + 3y = 8 (1) El tringulo con vrtices (1, 2), (3, 1), (4, 3) a e

64

4. Determinantes

Necesitaremos de las siguientes propiedades: Propiedad 12. (2) Si u, v R3 no cero, entonces u (u v) y v (u v) (3) u v = v u (4) En general no es cierto que u (v w) = (u v) w. (1) Si u, v son dos vectores no cero entonces u v = 0 u v

Demostracin. o (1) Tenemos que u v = ||u|| ||v|| cos(), donde es el angulo entre u y v. Por lo que u v = 0 cos() = 0. Debido a que el angulo entre vectores satisface 0 , se obtiene que = 0. k u (u v) = u a1 a2 a3 b1 b2 b3 =u = a1 a a a a a2 a3 1 3 + 1 2 k b1 b2 b1 b3 b2 b3 a a a a a2 a3 a2 1 3 + a3 1 2 b1 b2 b1 b3 b2 b3

(2) Pongamos u = (a1 , a2 , a3 ) y v = (b1 , b2 , b3 ). Calculemos u (u v):

= a1 (a2 b3 a3 b2 ) a2 (a1 b3 a3 b1 ) + a3 (a1 b2 a2 b1 ) = 0.

= a1 a2 b3 a1 a3 b2 a2 a1 b3 + a2 a3 b1 + a3 a1 b2 a3 a2 b1

Similarmente se prueba que v (u v) = 0 (Tarea).

1.4.1. Volumen de una caja. Supngase que v = (a 1 , a2 , a3 ), w = (b1 , b2 , b3 ), o u = (c1 , c2 , c3 ) son vectores en R3 . Desarrollaremos una frmula para calcular o el volumen V de la caja inclinada de la Figura 4. Se sabe que tal volumen es base por altura: V = Ah (25)

donde A es el area de la base y h es la altura de la caja. Tambin sabemos que e A = ||w u||. Ahora calculemos h. Si vemos tal caja de lado: (26)

1. Semntica geomtrica a e

65

z v
!d d

d u d d Dd y D d D d D d D D d D d x

d B d

d d d d d

Figura 4. Caja inclinada

T 

v h
E

E 

w donde P es un vector perpendicular tanto a w como a u: podemos suponer P = w u. Por lo que cos() = depejando h de sta ecuacin e o h = ||v|| cos() = pero (w u) v = ||w u|| ||v|| cos() donde es el angulo entre w u y v. Si P = w v, entonces (28) h >0 ||v||

||w u|| ||v|| cos() ||w u||

(27)

66

4. Determinantes

T 

v h
E

E 

c u w

por lo que + = , entonces = y as cos() = cos( )


1

= cos() cos() sin() sin()


0

= cos() = cos() Mientras que si P = w u entonces = y as cos() = cos(). En cualquier caso: | cos | = cos(). Por lo tanto, sustituyendo (29) en (27), h= y ahora sustituyendo (28), h= |(w u) v| ||w u|| ||w u|| ||v|| | cos()| ||w u|| (29)

juntando sta ecuacion con (25), y (26), se obtiene, que el volumen de la caja e inclinada es V = ||w u|| |(w u) v| ||w u|| = |(w u) v|

1. Semntica geomtrica a e

67

Podemos desarrollar ms, a condicin de escribir u = (c 1 , c2 , c3 ), w = (b1 , b2 , b3 ) a o y v = (a1 , a2 , a3 ). Entonces V = |(w u) v|

k = b1 b2 b3 (a1 , a2 , a3 ) c1 c2 c3 = |a1 (b2 c3 b3 c2 ) a2 (b1 c3 b3 c1 ) + a3 (b1 c2 b2 c1 )|

= |(b2 c3 b3 c2 , b1 c3 + b3 c1 , b1 c2 b2 c1 ) (a1 , a2 , a3 )|

Para simplicar la notacin se dene o Denicin 44. Si o a1 a2 a3 A = b1 b2 b3 c1 c2 c3 es matriz, entonces el determinante de A se dene como a1 a2 a3 b b b b b b det A = b1 b2 b3 = a1 2 3 a2 1 3 + a3 1 2 c1 c2 c1 c3 c2 c3 c1 c2 c3 Por lo que V = a1 a2 a3 b1 b2 b3 c1 c2 c3

Hemos demostrado: Teorema 11. Si v = (a1 , a2 , a3 ), w = (b1 , b2 , b3 ) y u = (c1 , c2 , c3 ) son vectores en R3 , entonces el volumen de la caja determinada por v, w y u es el valor absoluto del determinante de la matriz formada con v, w, u como las. Ejemplo 45. Hallar el determinante de la matriz 2 1 3 A = 4 1 2 1 2 2

68

4. Determinantes

Sol. 2 1 3 |A| = 4 1 2 2 1 2 =2

= 2(4) (8 4) + 3(8 2) = 2 + 12 + 24 = 34

1 2 4 2 4 1 (1) +3 1 2 2 2 2 2

Ejemplo 46. Hallar el volumen de la caja con vrtice en el origen detere minada por los vectores u = (4, 1, 1), v = (2, 1, 0), w = (0, 2, 3). Sol. El volumen est dado por el valor absoluto de a 4 1 1 2 1 2 0 1 0 + 2 1 0 =4 0 2 0 3 2 3 0 2 3 = 12 6 + 4 = 10 Tarea 20. Sean u, v, w R3 . Demuestre que u (v w) = (u w)v (u v)w Tarea 21. (1) Hallar el volumen de la caja que tiene los vectores dados como aristas adyacentes. (a) + 4 + 7 3 2 4 + 2 k, k, k 3 + 2 + 3 8hatk (b) 2 + 4k, k, (c) 3 + 4 2 + 7 5 3 + 10 k, k, k

(3) En los incisos siguientes usar un determinante para ver cuando los puntos dados estn en un plano en R3 (Sug: cul es el area de a a una caja con vrtices coplanares?). e (a) (0, 0, 0), (2, 1, 1), (3, 2, 1), (1, 2, 3) (b) (1, 2, 1), (3, 3, 4), (2, 2, 2), (4, 3, 5).

(2) En los incisos siguientes usar el determinante para ver cundo los a 2 . (Sugerencia: cul es puntos dados estn sobre una recta en R a a el area de un paralelogramos con vrtices colineales?): e (a) (0, 0), (3, 5), (6, 9) (b) (2, 3), (1, 4), (6, 2)

2. Caso general
Existen tambien determinantes para la matrices de orden mayor a tres.

2. Caso general

69

Denicin 47. Sea A una matriz n n: o a11 a12 a21 a22 A= . . . . . . an1 an2

ann

(1) La matriz menor i, j de A es la matriz denotada A i,j obtenida de A al eliminar el rengln i y la columna j. o

a1n a2n . . .

(2) El cofactor i, j de A es (1)i+j |Ai,j |. (3) El determinante de A es |A| = det A = a11 (1)1+1 |A1,1 | + a12 (1)1+2 |A1,2 | + a1n (1)1+n |A1,n | Como en el caso n = 2 y n = 3, el valor absoluto del determinante para n > 3 se puede interpretar como el volumen de una hipercaja inclinada en R n . Ejemplo 48. Hallar el cofactor 1, 2 2 3 A= 4 1 Sol. de 1 2 0 0 0 1 1 2 1 2 4 1

3 1 2 A1,2 = (1)1+2 4 1 4 1 2 1 = 3(7 + 14) = 21 Ejemplo 49. Calcular 5 2 4 1 0 1 5 2 1 2 0 1 3 1 1 1 Sol. 5 2 4 1 0 1 5 2 1 2 0 1 3 1 1 1 = 3 ((1 8) (4 4) + 2(8 1))

0 1 5 0 1 2 0 5 2 1 5 2 0 1 + 4 1 2 1 (1) 1 2 0 = 5 2 0 1 (2) 1 3 1 1 3 1 1 3 1 1 1 1 1

70

4. Determinantes

pero 1 5 2 2 0 1 = 1 5(1) + 2(2) = 8, 1 1 1 0 5 2 1 0 1 = 0(1) 5(1 + 3) + 2(1) = 22 3 1 1 0 1 2 1 2 1 = 0(2 1) 1(1 + 3) + 2(1 + 6) = 10 3 1 1 0 1 5 1 2 0 = 0(2 0) 1(1 0) + 5(1 + 6) = 36 3 1 1 det A = 5(8) + 2(22) + 4(10) + 1(36) = 8 La frmula dada en la denicin del determinante no es la unica forma de o o calcularlo. Teorema 12. Sea A = (aij ) matriz n n. Entonces (1) det A = ar1 cr1 + arn crn , 1 r n, donde cri es el cofactor r, i de A. (2) det A = a1s cis + ans cns , 1 s n. La frmula del primer inciso se llama desarrollo por menores en la r-sima o e la; mientras que al del segundo se llama desarrollo por menores en la ssima columna . e Por lo que un determinante se puede obtener de manter ja una la o una columna. Ejemplo 50. Hallar el determinante de 3 2 0 1 3 2 4 1 2 1 0 1 0 1 5 A= 1 2 0 1 2 0 0 0 0 2

por tanto,

2. Caso general

71

Sol. 3 2 0 1 2 4 1 2 , desarrollando por la 5a. la, 0 1 0 1 1 2 0 1

det A = 2(1)5+5

3 2 1 = 2(1)2+3 0 1 1 , desarrollando por la 3a columna, 1 2 1 = 2 3(1)1+1 = 12 Tarea 22. Calcular los determinantes de 2 6 2 1 5 2 1 1 0 6 1 1 4 , 4 1 1 , 0 4 1 , 2 4 0 0 5 5 0 1 3 0 2 0 1 2 6 2 4 , 0 4 1 1 1 2 2 2 5 2 0 3 4 6 0 9 6 1 0 2 1 1 0 2 1 1 1 + (1)(1)3+1 1 1 2 1 desarrollando por la 1a. columna, ,

1 3 6 0

0 4 1 1

1 1 0 2

0 1 2 4 8 1 1 1 1 2 2 5 4 5 1 2 0 1 2 4 1 8 1 5

Como puede notarse, siempre es buena idea buscar una columna o una la con la mayor cantidad de ceros posible. Desafortunadamente, no siempre se tiene una matriz con tantos ceros. Ejemplo 51. Calcular 2 1 3 2 0 1 6 1 3 7 3 2 5 0 4 8

72

4. Determinantes

Sol. Desarrollamos el determinante por la segunda la: 2 1 3 2 0 1 6 1 3 7 3 2 = 2 5 2 1 5 1 3 5 0 1 4 = 2(1)2+1 1 3 5 + 1(1)2+3 6 4 7 3 8 3 2 8 8 6 1 6 4 1 4 5 + 7 3 7 8 3 8 = 43

1 3 1 5 3 5 +5 3 3 2 3 8 2 8

3. Propiedades de los determinantes


Como ha podido notarse, el clculo de determinantes es un proceso claro, pero a engorroso. Se puede encontrar alivio si se usan las propiedades que vamos a exponer. Denicin 52. Sea A matriz de orden n m, o a11 a12 a1m . . . . . . A= . . . . an1 an2 anm

Propiedad 13. Si A es matriz n n, entonces det A = det At . Demostracin. Pongamos o a11 . A= . .

Se dene la matriz traspuesta de A como la matriz m n, a11 an1 a12 an2 At = . . . . . . a1m anm

an1

ann

Enseguida procedemos por induccin sobre n. Podemos tomar como caso o base n = 2. Entonces A= y det A = a11 a22 a12 a21 = det At a11 a12 , a21 a22 At = a11 a21 a12 a22

a1n . . .

3. Propiedades de los determinantes

73

Desarrollando el determinante de A por la primera la:

Supongamos ahora el resultado cierto para n; demostrmoslo para n + 1. e Tenemos a11 a12 a1n a1(n+1) . . . . . . . A= . . . . . a(n+1)1 a(n+1)2 a(n+1)n a(n+1)(n+1) det A = a11 |A1,1 | a12 |A1,2 | + + a1(n+1) (1)n+2 |A1,n+1 | por hip. de induccin, o pues (Ai,j )t = (At )j,i , = a11 |(A1,1 )t | a12 |(A1,2 )t | + + a1(n+1) (1)n+2 |(A1,n+1 )t |, = a11 |(At )1,1 | a12 |(A)t | + + a1(n+1) (1)n+2 |(At )n+1,1 |, 2,1 = det At , desarollando por la primera columna.

En vista de sta propiedad, cualquier propiedad que se cumpla para el dee terminante por las, se cumplir para columnas, y rec a procamente. Propiedad 14. Sea A una matriz n n. Si B es la matriz obtenida de A por un intercambio de las entonces det B = det A. Demostracin. Por induccin sobre n. El caso base: si n = 2 y o o A= entonces B= y tenemos que det B = a21 a12 a22 a11 = det A. = (a11 a22 a12 a21 ) a11 a12 a21 a22 a21 a22 a11 a12

Supongamos la propiedad para matrices de orden n n y hagamos el paso inductivo. Ponemos a11 a12 a1n a1(n+1) . . . . . . . A= . . . . . a(n+1)1 a(n+1)2 a(n+1)n a(n+1)(n+1)

74

4. Determinantes

y suponemos que B es la matriz que se obtiene de A intercambiando las las i y j con i < j: a11 a12 a1n a1(n+1) . . . . . . . . . . . . aj1 aj2 ajn aj(n+1) . . . . . . . B= . . . . . ai1 ai2 ain ai(n+1) . . . . . . . . . . . . a(n+1)1 a(n+1)2 a(n+1)n a(n+1)(n+1)

Notemos que si k = i y k = j entonces las matrices menores B k,r , Ak,r son de orden n n y una se obtiene de otra por un intercambio de las; luego, por hiptesis de induccin |Ak,r | = |Bk,r |, 1 r n + 1 y as desarrollando o o , det B por una la k con k = i y k = j:
det B = ak1 (1)k+1 |Bk,1 | + ak2 (1)k+2 |Bk,2 | + ak(n+1) (1)k+n+1 |Bk,n+1 | = ak1 (1)k+1 (1)|Ak,1 | + ak2 (1)k+2 (1)|Ak,2 | + ak(n+1) (1)k+n+1 (1)|Ak,n+1 | = (1) det A

= (1) ak1 (1)k+1 |Ak,1 | + ak2 (1)k+2 |Ak,2 | + ak(n+1) (1)k+n+1 |Ak,n+1 |

pues la ultima frmula corresponde al desarrollo por menores de det A en o la k la.

Corolario 1. Si A es una matriz nn con dos las iguales entonces det A = 0. Demostracin. Supongamos que A tiene la forma o . . . . . . ai1 ain . . . A= . . . ai1 ain . . . . . .

entonces al intercambiar tales las obtenemos la misma A. Pero por la propiedad inmediata anterior, det A = det A, lo que implica det A = 0.

3. Propiedades de los determinantes

75

Propiedad 15. Si a11 . . . A = ai1 . . . an1 entonces

an1 . . . ai1 , . . . ann

a11 . . . B = rai1 . . . an1

an1 . . . rai1 . . . ann

detB = r det A. Demostracin. Primero hagamos notar que en las matrices menores A i,j o y Bi,j la la que hace diferente a A de B se elimina, por lo que A i,j = Bi,j , 1 j n. Desarrollando el determinante de B por menores en la i la: det B = rai1 (1)i+1 |Bi,1 | + rai2 (1)i+2 |Bi,2 | + + rain (1)i+r |Bi,n | = rai1 (1)i+1 |Ai,1 | + rai2 (1)i+2 |Ai,2 | + + rain (1)i+r |Ai,n | = r ai1 (1)i+1 |Ai,1 | + ai2 (1)i+2 |Ai,2 | + + ain (1)i+r |Ai,n | = r det A.

Ejemplo 53. Calcular el determinante de 2 1 3 4 6 2 1 4 A = 6 3 9 12 2 1 3 4 1 4 2 1

Sol. Podemos notar que los registros de la tercera la tienen factor com n 3, por lo que u 2 6 det A = 3 2 2 1 1 2 1 1 4 3 1 3 3 2 4 4 4 4 1 2 1 2 1 1

2 1 6 1 1

pero entonces la primera la se repite en la tercera. Por lo que det A = 0.

76

4. Determinantes

Propiedad 16. Supngase que A es una matriz o obtenida de A multiplicando una la de A por un la a otra. Es decir, a11 a1n a11 . . . . . . . . . ai1 ain ai1 . . , B = . . . A= . . . . aj1 ajn aj1 + rai1 . . . . . . . . . an1 ann an1 Entonces det A = det B

n n y B es la matriz nmero y sumndo sta u a e a1n . . .

ain . . . . ajn + rain . . . ann

Demostracin. Desarrollamos el determinante de B por menores en la o jsima la: e det B = (aj1 + rai1 )(1)j+1 |Bj,1 | + + (ajn + rain )(1)j+n |Bj,n | pues |Aj,k | = |Bj,k |,

= (aj1 + rai1 )(1)j+1 |Aj,1 | + + (ajn + rain )(1)j+n |Aj,n |, = aj1 (1)j+1 |Aj,1 | + + ajn (1)j+n |Aj,n |

+ rai1 (1)j+1 |Aj,1 | + + rain (1)j+n |Aj,n | distribuyendo, = det A + r(ai1 (1)i+1 |Aj,1 | + + ain (1)j+n |Aj,n |) . . . . . . ai1 ain . , . . = det A + r . . . ai1 ain . . . . . . donde la la repetida aparece en la jsima la, e = det A + 0 = det A.

Propiedad 17. Si A es matriz nn triangular superior o inferior, entonces det A es el producto de sus elementos diagonales.

3. Propiedades de los determinantes

77

Demostracin. Caso: A triangular superior: tenemos o

a11 0 a22 A= . . . . . . 0 0

. . . ann

Shora procedemos por induccin sobre n. Si n = 2, entonces det A = a 11 a22 o y el resultado se cumple. Si suponemos el resultado cierto para matrices de orden (n 1) (n 1) entonces, desarrollando det A por menores en la primera columna:

det A = a11 |A1,1 |

= a11 (a22 . . . ann )

Caso: anterior.

pues A1,1 es una matriz (n 1) (n 1) e hiptesis de induccin. o o

A matriz triangular inferior: se procede similarmente al caso

Ejemplo 54. Calcular el determinante de A, donde

2 1 3 2 0 1 A= 6 1 3 7 3 2

5 0 4 8

78

4. Determinantes

Sol.

det A = det At 2 1 = 3 5 4 1 = 3 5 2 0 1 0 0 0 1 0
3+2

6 7 1 3 3 2 4 8 0 3 1 3 , 3 2 4 8 4 0 3 1 1 3 , 5 4 8 3F (2) + 1F ;

= 1(1)

desarrollando por la 2a. columna,

4 0 3 = 1 1 3 , 9 0 4 = 4 3 , 9 4

2F (4) + 3F, desarrollando por la 2a. columna,

= (16 + 27) = 43

Ejemplo 55. Calcular el determinante

2 3 0 2

2 3 1 0

0 2 3 2

4 2 2 1

3. Propiedades de los determinantes

79

Sol.

2 3 0 2

2 3 1 0

0 2 3 2

4 2 =2 2 1

1 3 0 2

1 3 1 0

0 2 3 2

2 2 2 1 0 2 2 4 , 3 2 2 3 1F (3) + 2F, 1F (2) + 4F

1 1 0 0 =2 0 1 0 2

1 1 0 1 = 2 0 0 0 2 1 0 = 2 0 0 1 1 0 0

0 2 3 2 , 2 4 8 1 1 1 0 0

0 2 3 2 , 2 4 2 3

2F 3F

2F 2 + 4F

1 0 = 2 2 0 0 1 1 0 1 = 4 0 0 0 0 = 4 17 = 68

0 2 3 2 1 2 8 1 3F (8) + 4F

0 4 3 2 , 1 5 0 17

Ejemplo 56. Calcular

1 1 1 1

a b c d

b c+d c a+d d a+b a b+c

80

4. Determinantes

Sol. 1 a 1 b 1 c 1 d

1 a b c+d 1 b c a+d = 1 c d a+b 1 d a b+c

b c d a

a+b+c+d b+c+a+d , c+d+a+b d+a+b+c a b c d b c d a 1 1 1 1

2C + 4C, 3C + 4C;

1 1 = (a + b + c + d) 1 1 =0 Ejemplo 57. Calcular

1 a a2 1 b b2 1 c c2 Sol. 1 a a2 1 a a2 2 = 0 a b a 2 b2 , 1 b b 1 c c2 0 a c a 2 c2

1F (1) + 2F, 1F (1) + 3F,

1 a a2 = 0 a b (a b)(a + b) 0 a c (a c)(a + c)

1 a a2 = (a b)(a c) 0 1 a + b 0 1 a+c 1 a a2 = (a b)(a c) 0 1 a + b , 0 0 bc = (a b)(a c)(b c) 2F (1) + 3F,

Tarea 23. Calcular (1) 1 4 9 4 9 16 9 16 25 16 25 36 (2) a b c 3a 2b 3b + 2c 3c + 2a b c a 16 25 36 49

3. Propiedades de los determinantes

81

(3) 1 a b+c 1 b a+c 1 c a+b (4) 1 + x 1 y1 1 + x 1 y2 1 + x 1 y3 1 + x 1 y4 (5) 0 a b c a 0 e f b e 0 h c f h 0 d g k l x 1 1 x . . . . . . 1 1 (7) a1 x x a2 . . . . . . x x x x . . . an 1 1 1 1 . . . . . . 1 x d g k l 0 1 + x 2 y1 1 + x 2 y2 1 + x 2 y3 1 + x 2 y4 1 + x 2 y1 1 + x 2 y2 1 + x 2 y3 1 + x 2 y4 1 + x 4 y1 1 + x 4 y2 1 + x 4 y3 1 + x 4 y4

(6) El determinante n n

Recordemos que el mtodo de Gauss se basa en las siguientes operaciones e elementales por las: (1) intercambio de las; (2) multiplicacin de una la por un nmero diferente de cero; o u (3) multiplicacin de una la por un nmero y su suma a otro la. o u Lema 1. Una matriz A de orden nm puede llevarse a una matriz escalonada usando solamente las operaciones elementales 1 y 3. Demostracin. Pongamos o a11 . A= . . a1m . . .

an1

anm

82

4. Determinantes

entonces hay dos casos: A = 0 o A = 0. Si lo primero A es triangular superior y se cumple el lema trivialmente. Si A es no cero entonces debe de tener una primera columna con no todas sus entradas cero: supongamos que tal es la jsima columna: e 0 . . A = . 0 a1j . . . . . . 0 anj a1m . . .

anm

podemos suponer adems que aij = 0: a 0 . . . A = 0 . . . 0 0 a1j . . . . . . 0 aij . . . . . . a1m 0 0 aij . . . . . . . . . . . . i1 0 0 a1j aim . . . . . . . . . . . . anm 0 0 anj 0 0 aij aim 0 0 0 b2m . . . . . . . . . . . . 0 0 a1j a1m . . . . . . . . . . . . 0 0 anj anm 0 0 aij aim 0 0 0 b2m 0 0 0 b3m . . . . . . . . . . . . 0 0 a1j a1m . . . . . . . . . . . . 0 0 anj anm 0 aij aim 0 0 b2m 0 0 b3m . . . . . . . . . 0 0 bim . . . . . . . . . 0 0 bnm aim . . . a1m . . . anm

0 anj

1F (a2j /aij )+2F

1F (a2j /aij )+2F

0 0 0 . . . 0 . . . 0

3. Propiedades de los determinantes

83

siendo sta ultima una matriz escalonada. e

y de nuevo, hasta obtener 0 0 A . . .

Ahora repetimos el procedimiento anterior en la submatriz b(2(j+1) b2m . . . B= . . . bn(j+1) bnm 0 0 0 . . . . . . 0 . . . . . .

Teorema 13. Si A, B son matrices n n entonces det(AB) = det A det B Ejemplo 58. Si A es matriz n n tal que det A = 3, hallar det A 5 . Sol. det A5 = det(AAAAA) = det A det A det A det A det A = 35 = 243 Denicin 59. Una matriz A de orden n n se llama invertible si existe o B matriz tal que (1) AB = In ; (2) BA = In . En tal caso B se llama matriz inversa de A y se denota como A 1 . Ejemplo 60. Sea 1 3 . 0 2 Encontrar, si es que existe, su matriz inversa. A= Sol. Tenemos que encontrar una matriz B que cumpla con 1 3 B= 0 2 Pongamos x y . z w Entonces la primera de las ecuaciones de (30) es equivalente a B= x + 3z y + 3w 2z 2w = 1 0 0 1 1 0 , 0 1 B 1 3 0 2 = 1 0 0 1 (30)

84

4. Determinantes

que es equivalente al sistema de ecuaciones x y + 3z = + 3w = 2z = 2w = 1 0 0 1

de donde w = 1/2, z = 0, y = 3/2, x = 1. Veriquemos, ahora que B= 1 3 0 1/2

sistema que puede ser resuelto 1 0 3 0 0 1 0 3 0 0 2 0 0 0 0 2

por el mtodo de Gauss e 1 0 3 0 1 1 0 0 1 0 3 0 0 0 0 1 0 0 1 0 0 0 1 1 2

es la inversa de A, es decir, tenemos que checar AB = I 2 y BA = I2 : AB = = y adems a BA = = Por lo tanto, A1 = 1 3/2 . 0 1/2
3 1 2 1 0 2

1 3 0 2 1 0 0 1

1 3 2 0 1 2

1 3 0 2

1 0 . 0 1

Es ms fcil calcular la matriz inversa si se utiliza la frmula dada por la a a o tarea siguiente. Tarea 24. Sea a b c d matriz con det A = 0. Demostrar que A tiene como matriz inversa a A= d/ det A b/ det A c/ det A a/ det A No toda matriz tiene inversa:

3. Propiedades de los determinantes

85

Ejemplo 61. Demostrar que la matriz 1 1 2 2 no tiene inversa. Sol. Razonaremos por contradiccin. Supongamos que tal matriz si o tiene inversa llamada B. Entonces 1 0 1 1 B= 0 1 2 2 sacando determinante a ambos lados de la igualdad det 1 0 1 1 det B = det 0 1 2 2
0

=1

lo cual implica que 0 = 1: un absurdo. Se concluye que la matriz 1 1 2 2 no tiene inversa. Nos encaminamos a demostrar que la condicin suciente y necesaria para o que una matriz tenga inversa es que su determinante no sea cero. Propiedad 18. Si una matriz A tiene inversa entonces det A = 0 y adems a 1 = 1/ det A. det A Demostracin. Tenemos que AA1 = In . Sacando determinante a ambos o lados de sta ecuacin obtenemos que det A det A 1 = 1, por que no es e o posible que det A = 0. Despejando det A 1 , se obtiene 1 det A1 = . det A

Lema 2. Si

y ci,j =

(1)i+j |A

i,j |

es el cofactor i, j de A entonces
n

a11 . A= . .

an1

ann

a1n . . .

aik cj,k =
k=1

det A, 0,

si i = j; si i = j.

(31)

86

4. Determinantes

y
n

cl,i alj =
l=1

det A 0

si i = j; si i = j.

(32)

Demostracin. Primero demostraremos la ecuacin (31). Tomemos i, j o o con 1 i, j n. Entonces hay dos casos i = j o bien i = j. En el primer caso: a11 a12 a1n . . . . . . . . . ai1 ai2 ain 0= . . . , repetimos la ila en la jla, . . . . . . ai1 . . .
n

ai2 . . .

ain . . .

= ai1 cj,1 + ai2 cj,2 + ain cjn , por menores en la jla =


k=1

aik cj,k .

Y si i = j, entonces desarrollando el determinante de A por la ila, det A = ai1 ci,1 + ai2 ci,2 + ain ci,n
n

=
k=1

aik ci,k .

En resumen, se cumple la ecuacin (31). o Tarea 25. Vericar la ecuacin (32). o

Denicin 62. Si A es una matriz n n y c i,j es el cofactor i, j de A o entonces, la matriz adjunta de A es t c1,1 c1,n . . . adj A = . . . cn,1 cn,n La matriz adjunta se usa para calcular la matriz inversa, como veremos en el siguiente teorema. Teorema 14. Sea A matriz n n, entonces

3. Propiedades de los determinantes

87

(1) (adj A)A = (det A) In ; (2) A(adj A) = (det A) In . Demostracin. o (1) c1,1 . (adj A)A = . . cn,1 c1,1 . = . . c1,n
n

cn,n

t c1,n . A . .

cn,n

a11 cn,1 . . . . . .

an1

ann

a1n . . .

=(
l=1

cl,i alj )i,j 0 0 . . . det A

= (det A) In (2) Tarea.

det A 0 0 0 det A 0 = . . . . . . . . . 0 0 0

u , seg n el lema inmediato anterior.

Corolario 2. Si A es matriz n n y det A = 0, entonces A tiene inversa y A1 = 1 adj A det A (33)

Ejemplo 63. Hallar la inversa de 4 0 1 A = 2 2 0 . 3 1 1 Sol. Primero calculamos el determinante de A:

88

4. Determinantes

4 0 1 |A| = 2 2 0 3 1 1 = 4 0 1 2 2 0 , 1 1 0 1F (1) + 3F ;

= (1)1+3 =4 Los cofactores son c1,1 = (1)2 c2,1 = (1)3 c3,1 = (1)4 Entonces, 2 0 = 2, 1 1 0 1 = 1, 1 1 0 1 = 2, 2 0

2 2 1 1

c1,2 = (1)3 c2,2 = (1)4

2 0 = 2, 3 1 4 1 = 1, 3 1 4 1 = 2, 2 0

c1,3 = (1)4 c2,3 = (1)5

4 0 = 4 3 1

2 2 = 4 1 1

c3,2 = (1)5

c3,3 = (1)6

4 0 = 8. 2 2

y as seg n la frmula (33), , u o A1

t 2 1 2 2 2 4 2 1 4 = 2 1 adj A = 1 4 4 6 2 2 8 1/2 1/4 1/2 = 1/2 1/4 1/2 1 1 2

Tarea 26. Hallar A1 si A es invertible para cuando (1) A= (2) A= (3) 2 1 1 A = 0 1 1 2 1 1 2 0 1 1 4 1 2 1

4. Regla de Cramer

89

(4) 3 0 4 A = 2 1 1 3 1 2 2 1 3 A = 0 1 4 1 2 1

(5)

4. Regla de Cramer
Un sistema de ecuaciones a11 x1 + a12 x2 + + a1m xm = b1 . . .

an1 x1 + an2 x2 + es equivalente a la ecuacin matricial o a11 a12 . . . . . . an1 an2

+ anm xm = bn

As un sistema de ecuaciones puede escribirse como , AX = b x1 b1 . . y b = . . donde A = (aij ), X = . . . xm bn

b x1 a1m 1 x2 b2 . . . = . . . . . . . anm bn xm

Teorema 15 (Regla de Cramer). Sea un sistema de n ecuaciones con n incgnitas o AX = b (34) donde A = (aij ), x1 . X = . , . xn b1 . b = . . . bn

Si det A = 0 entonces el sistema (34) tiene como soluciones xk = det Bk , det A k = 1, . . . , n

90

4. Determinantes

donde Bk es la matriz que se obtiene de A al reemplazar la k-sima columna e de A por el vector columna b: k a11 a12 . . . Bk = . . . an1 an2 b1 . . .
k

ain . . .

bn

ann

Demostracin. Si AX = b entonces o

X = A1 b 1 adj Ab. = det A Entonces x1 c11 c21 cn1 b1 x2 1 . . . . . . . . = . . . . . . det A . bn c1n c2n cnn xn 1 det A (c11 b1 + c21 b2 + cn1 bn ) 1 = det A (c12 b1 + c22 b2 + cn2 bn ) . . .
1 det A

(c1n b1 + c2n b2 + cnn bn )

lo que implica

xk =

1 (b1 c1k + b2 c2k + + bn cnk ) det A


k

1 = det A

a11 . . .

a12 . . .

b1 . . . bn
k

ain . . . ann

an1 an2

desarrollando el determinante por la ksima columna. e

Ejemplo 64. Resolver por la regla de Cramer el sistema 5x1 2x2 + x3 = 1 3x1 + 2x2 = 3 x1 + x 2 x 3 = 0

4. Regla de Cramer

91

Sol. La matriz de coecientes del sistema es 5 2 1 0 A = 3 2 1 1 1 cuyo determinante es 5 2 1 0 |A| = 3 2 1 1 1

5 2 1 = 3 2 0 , 6 1 0 = 15

1F + 3F,

luego, por la regla de Cramer 1 2 1 1 3 2 0 , 15 0 1 1 5 1 1 1 3 3 0 , 15 1 0 1 5 2 1 1 3 2 3 15 1 1 0

x1 = pero

x2 =

x3 =

1 2 1 1 2 1 3 2 0 = 3 2 0 , 0 1 1 1 1 0 = (1)1+3 = 5, adems a 5 1 1 5 1 1 3 3 0 = 3 3 0 , 6 1 0 1 0 1 = (1)4 =3 1 1 6 1 = 15

1F + 2F,

3 2 1 1

1F + 3F

3 3 6 1

92

4. Determinantes

y 5 2 1 5 2 1 3 2 3 = 12 8 0 1 1 0 1 1 0 = (1)1+3 = 20 por lo tanto x1 = 5 20 , x2 = 1, x3 = 15 15 12 8 1 1

Podemos comparar con Maxima:


Maxima

solve([5*x[1]-2*x[2]+x[3]=1,3*x[1]+2*x[2]=3,x[1]+x[2]-x[3]=0], [x[1],x[2],x[3]]);
TEX Output

x1 =

4 1 , x2 = 1, x3 = 3 3

Tarea 27. (1) Resolver por la regla de Cramer (cuando sea posible): (a) x1 2x2 = 1 3x1 + 4x2 = 3 (b) 3x1 + x2 = 5 2x1 + x2 = 0 (c) x1 + 2x2 x3 = 2 2x1 + x2 + x3 = 0 3x1 x2 + 5x3 = 0 (d) 3x1 + 2x2 x3 = 1 x1 4x2 + x3 = 2 5x1 + 2x2 = 0

4. Regla de Cramer

93

(e) 5x1 2x2 + x3 = 1 x2 + x 3 = 0 x1 + 6x2 x3 = 4 (2) Hallar x2 del sistema (a) x1 + x2 3x3 2x1 + x2 x2 x 3 3x1 + x2 (b) 6x1 x1 x1 x1 + x2 x3 x2 + 5x4 + 3x2 + x3 + x2 x3 + 2x4 = 4 = 2 = 2 = 0 + x4 + 2x4 x4 + x4 = = = = 1 0 5 1

Cap tulo 5

Nmeros complejos u

Al conjunto de nmeros complejos se les denota con C y ste es el siguiente u e C = {(a; b) | a, b R} es decir, el conjunto de nmeros complejos es el conjunto de parejas de nmeros u u reales. En particular sto signica que un slo nmero complejo consta de dos e o u nmeros reales. u Ejemplos 65. (1) (1; 3) C (2) ( 2; 1) C
3 (3) ( 1 ; 2 ) C 2

Denicin 66 (Igualdad). Dos nmeros complejos (a; b) y (c; d) se dice que o u son iguales escribindose e (a; b) = (c; d) a = c b = d Ejemplo 67. Se puede escribir 1 ;2 3 2 = 2 ; 12 2

porque 1/ 2 = 2/2 y 2 3 = 12. 95

96

5. N meros complejos u

Los nmeros complejos se pueden sumar y multiplicar de la siguiente forma: u Denicin 68 (suma y producto). Sean (a; b), (c; d) C; o (1) (a; b) + (c; d) = (a + c; b + d) (2) (a; b)(c; d) = (ac bd; ad + bc)

1. Suma y producto
Las propiedades de la suma y de la multiplicacin compleja son las usuales. o Propiedad 19. Si u, v, w C entonces (1) u + v = v + u (conmutativa) (2) (u + v) + w = u + (v + w) (asociativa) (3) uv = vu (conmutativa) (4) (uv)w = u(vw) (asociativa) (5) u(v + w) = uv + uw (distributiva,factorizacin) o Demostracin. o (1) Tarea (2) Tarea (3) Pongamos u = (a; b) y v = (c; d). Entonces uv = (ac bd; ad + bc). Por otro lado vu = (ca db; cb + da) = uv, = (ac bd; ad + cb), por las propiedades conmutativas en R; seg n (35). u (35)

(4) Tarea. (5) Tarea.

La forma de escribir los nmeros complejos como paraejas de nmeros reales u u separados por un punto y una coma no es la ms popular. La forma que es la a que es usual es la llamada forma binmica. o

1. Suma y producto

97

Denicin 69. o (1) Si u = (a; b) C entonces se escribe u = a + bi sta forma se llama binmica. e o (2) Si a R, entonces a se considera que es un nmero complejo esu cribiendo a = a + 0i = (a; 0) (3) Si b R se escribe ib = 0 + ib = (0; b) y tal nmero complejo se llama complejo puro. u

Ejemplos 70. Siguiendo las deniciones se obtiene que: (1) i = (0; 1) (2) 1 = (1; 0) = 1 + 0i (3) 0 = (0; 0).

Ntese que como consecuencia de las deniciones podemos escribir o RC Es fcil demostar las siguientes propiedades a Propiedad 20. Si z C; (1) 1z = z; (2) 0z = 0; (3) 0 + z = z.

Tambis es trivial la demostracin del siguiente teorema. Sin embargo, es e o la ms importante propiedad de los nmeros complejos. a u Teorema 16. i2 = 1

98

5. N meros complejos u

Demostracin. o i2 = ii = (0; 1)(0; 1) = (0 0 1 1; 0 1 + 1 0) = (1; 0) = 1

Todos los calculos que se hagan con los nmeros complejos se basarn en u a el teorema 16. Adems, en general, lo que se busca en cada calculo es la forma a binmica. o Ejemplo 71. Hallar (1 + i)3 Sol. (1 + i)3 = (1 + i)(1 + i)2 = (1 + i)(12 + 2i + i2 ) = (1 + i)2i = 2i + 2 i2 = 2 + 2i, concluimos que (1 + i)3 = 2 + 2i Tambin se pudo haber usado el desarrollo de un binomio cbico: e u (1 + i)3 = 13 + 3(1)2 i + 3(1)i2 + i3 = 1 + 3i + 3(1) + i i2 = 2 + 3i + i(1) = 2 + 2i
1 1 1

1. Suma y producto

99

Ejemplo 72. Reducir a la forma binmica el cubo de o 3 1 = +i 2 2 Sol. Tenemos que 3 1 3 3 = +i 2 2 2 1 3 1 3 = +i +i 2 2 2 2 1 1 3 3 23 = +i i +i 4 2 4 2 2 1 1 3 3 = i +i 2 2 2 2 3 1 3 1 = i i + 2 2 2 2 2 1 2 i 3 , diferencia de cuadrados, = 2 2 2 1 3 = i 2 2 4
2

3 1 = 4 4 =1 En Maxima el nmero i se llama como %i. En general, todas las constantes u matemticas en Maxima se llaman anteponindoles el s a e mbolo %. Por ejemplo el nmero e es %e: el nmero 3.1416 es %pi. O u u
Maxima
Initial

omega:-1/2+%i*sqrt(3)/2;
TEX Output

3i 1 2 2

100

5. N meros complejos u

Maxima

omega^3;
TEX Output

3i 1 2 2

le pedimos al sistema que reduzca sta expresin con expand(%). Aqu el e o s mbolo de porcentaje es para indicar la instruccin inmediata anterior. o
Maxima

expand(%);
TEX Output

2. Restas y divisiones
La forma de restar en C es como en R2 . En cuanto a la divisin su forma es o ms complicada. En la denicin siguiente se dan las frmulas. No signica a o o que el lector tenga que aprenderlas de memoria. Lo unico que debe de terner en cuenta el lector es que en C se cumplen las propiedades algebraicas usuales y que i2 = 1. Denicin 73. Si u = (a; b) y v = (c; d) en C entonces se denen o (1) u v = (a c; b d) (2) Si v = 0 u = v ac + bd bc ad ; c2 + d 2 c2 + d 2

Ejemplo 74. Reducir 1+i 1i

2. Restas y divisiones

101

Sol. La idea es multiplicar numerador y denominador por el conjugado del denominador: 1+i (1 + i) (1 + i) = 1i (1 i) (1 + i) (1 + i)2 = 1 i2 1 + 2i + i2 = 1 (1) 2i = 2 =i

Ejemplo 75. Reducir (3 + 2i)2 (1 3i) (3 + i)2 (1 + 2i) Sol. Primero desarrollamos los cuadrados: (3 + 2i)2 (1 3i) (9 + 12i + 4i2 )(1 3i) = (3 + i)2 (1 + 2i) (9 + 2i + i2 )(1 + 2i) (9 4 + 12i)(1 3i) = (9 1 + 2i)(1 + 2i) (5 + 12i)(1 3i) = (8 + 2i)(1 + 2i) 5 15i + 12i 36i2 = , distribuyendo 8 + 16i + 2i + 4i2 5 + 36 3i = 8 4 + 18i 41 3i = 4 + 18i (41 3i) (4 18i) = (4 + 18i) (4 18i) (41 3i)(4 18i) = 42 182 i2 164 738i 12i + 18i2 = 340 146 750i = 750 73 = i 375

102

5. N meros complejos u

Maxima

Initial

(3+2*%i)^2*(1-3*%i)/((3+%i)^2*(1+2*%i));
TEX Output

(1 3i) (2i + 3)2 (i + 3)2 (2i + 1)

Maxima

expand(%);
TEX Output

3i 41 22i 4 22i 4

Maxima

factor(%);
TEX Output

3i 41 2 (11i 2)

Tarea 28. (1) Reducir (a) (7 i) + (6 + 3i) (4 + 3i) (b) (2 3i)i (c) (2 + i)(1 + 2i) (d) 1/i (e) (1 + i)/i (f) 1+i i + i 1i (g) (2 + i)(1 2i) 3i

2. Restas y divisiones

103

(h) (4 + 3i)(1 2i) 7i 1+i 2 (j) i 1+i+


i i 1+i+ 1+i 4

(i)

(2) Hallar los valores de x, y R tales que satisfacen

(3) Hallar los x R tales que (a) (1 + i)x3 + (1 + 2i)x2 (1 + 4i)x 1 + i = 0 (b) (1 + i)x3 + (1 + 2i)x2 (1 + i)x i 2i = 0 Denicin 76. Si z = a + ib C se dene o (1) su conjugado como (2) su mdulo como o (3) la parte real como z = a ib |z| = a2 + b2 z=a (4) la parte imaginaria como z=b

(1 + i)(x + 2y) (3 2i)(x y) = 8 + 3i

Ejemplos 77. (1) 1 + i = 1 i (2) = i i (3) = 3 3 (4) |1 i| = 1+ (1)2 = 2

(7)

(6) |i| = 1 (8)

(5) 1 i = 1 + i (1 i) = 1; 3 = 3; (1 i) = 1

3=0

(9) |3| = 3; | 3| = 3.

104

5. N meros complejos u

Propiedad 21. Si z, z1 , z2 C (2) z1 z2 = z1 z2 (3) z1 + z2 = z1 + z2 (5) |z1 z2 | = |z1 | |z2 | Demostracin. o (1) Podemos poner z = a + bi con a, b R. Entonces zz = (a + bi)(a bi) = a 2 b 2 i2 = a2 + b2 =( (2) Tarea (3) Tarea (4) Pongamos z = a + bi. Entonces z = z a + bi = a bi b=0 (5) Tenemos que |z1 z2 |2 = z1 z2 (z1 z2 ), por (1) = z1 z2 z1 z2 , por (2) = z 1 z1 z2 z2 = |z1 |2 |z2 |2 , por (1) es decir, ahora, sacando ra cuadrada a ambos lados de la ecuacin y como z o el mdulo es no negativo, obtenemos o |z1 z2 | = |z1 | |z2 |. |z1 z2 |2 = |z1 |2 |z2 |2 a = a b = b, por igualdad de complejos, (4) z = z z R (1) |z|2 = zz

(6) |z1 + z2 | |z1 | + |z2 |

a 2 + b 2 )2

= |z|2

z=aR

3. Representacin geomtrica o e

105

3. Representacin geomtrica o e
Si z = a + bi C entonces z se puede representar como un punto o un vector el plano:
T

b |z|

a Tal plano se llama el plano complejo; el eje y se llama eje imaginario y el eje x se llama eje real; el angulo se llama argumento de z y se denota como arg z. Ntese que |z| es la longitud de z: o
T

d  |z| d

a Obsrvese que e cos = de donde se obtiene a , |z| sin = b |z|

z = a + ib = |z| cos + i|z| sin = |z|(cos + i sin ) es decir z = |z|(cos + i sin ) (36)

106

5. N meros complejos u

El lado derecho de la ecuacin (36) se llama forma trigonomtrica o forma o e polar de z. Ejemplo 78. Escribir en forma trigonomtrica los n meros siguientes. e u (1) z = 1 + i (2) z = 1 Sol. (1) Dibujemos:
T

1+i

/4

1 entonces, por denicin o 1 + i = |1 + i|(cos(/4) + i sin(/4)) = 2 (cos(/4) + i sin(/4)) (2) Dibujamos de nuevo:
T

por lo que 1 = | 1|(cos + i sin ) cos + i sin

3. Representacin geomtrica o e

107

En Maxima la instruccin para conseguir la forma trigonomtrica es polarform. o e Por ejemplo


Maxima

polarform(1+%i);
TEX Output

i 2e 4

La expresin ei/4 es una abreviatura para cos(/4) + i sin(/4) segn la o u denicin de Euler: o Denicin 79 (Euler). Si R o

ei = cos + i sin

Podemos entonces escribir la forma trigonomtrica de un nmero complejo e u como z = |z|ei arg z . (37)
Maxima

polarform(-1);
TEX Output

ei

Si se quiere evitar el uso de las exponenciales se puede poner demoivre:true;


Maxima
Initial

demoivre:true;
TEX Output

true

108

5. N meros complejos u

Maxima

polarform(1+%i);
TEX Output

2i 2 + 2 2

sin embargo, a veces tal resulta en identidades tautolgicas: o


Maxima

polarform(-1);
TEX Output

restablecemos la identidad de Euler:


Maxima

demoivre:false;
TEX Output

false

Maxima

polarform(-1);
TEX Output

ei

de donde se puede deducir el argumento de -1: .

3. Representacin geomtrica o e

109

Tarea 29. Dibujar en el plano complejo y escribir los siguientes nmeros u en forma trigonomtrica: e (2) i (1) i

(4) 1 + 2i (6) 1/2 + 2i (7) 1/2 + i 3/2 (8) 6i 3.1. Frmula de Moivre. Debemos recordar las frmulas para el seno y el o o coseno de la suma de dos angulos: sin( + ) = sin() cos() + sin() cos() cos( + ) = cos() cos() sin() sin() (38) (39) (5) 3 2i

(3) 1 i

Teorema 17. Si z1 , z2 C tales que su escritura en forma trigonomtrica e son z1 = r1 (cos + i sin ) z2 = r2 (cos + i sin ) entonces (1) z1 z2 = r1 r2 (cos( + ) + i sin( + )) (2) r1 z1 = cos( ) + i sin( ) z2 r2

Demostracin. o (1) z1 z2 = r1 r2 (cos + i sin )(cos + i sin ) = r1 r2 (cos cos sin sin + i(sin cos + cos sin ) = r1 r2 (cos( + ) + i sin( + )) usando las ecuaciones (38) y (39). (2) Anloga al anterior. a

110

5. N meros complejos u

Corolario 3 (frmula de Moivre). Si n Z; o

(cos + i sin )n = cos n + i sin n.

Ejemplo 80. Expresar cos 3 en trminos de cos . e Sol. Por la frmula de Moivre o (cos + i sin )3 = cos 3 + i sin 3 Por el teorema del binomio de Newton, el lado izquierdo de la ecuacin es o (cos + i sin )3 = cos3 + 3 cos2 i sin + 3 cos i2 sin2 + i3 sin3 = cos 3 cos sin + i 3 cos sin sin
3 2 2 1 3 i

es decir

lo que implica, por denicin de la igualdad entre n meros complejos, que o u sus partes reales e imaginarias deben de ser iguales entre s . cos 3 = cos3 3 cos sin2 = cos3 3(cos )(1 cos2 ) = cos3 3 cos + 3 cos3 = 4 cos3 3 cos .

cos3 3 cos sin2 + i 3 cos2 sin sin3 = cos 3 + i sin 3

Concluimos que

cos3 = 4 cos3 3 cos . Tarea 30. (1) Expresar sin 3 en trminos de sin . e (2) Expresar cos 4 en trminos de cos . e (3) Expresar sin 4 en trminos de sin . e Sabemos que la ecuacin o x2 = 2 tiene soluciones en R dadas por x = 2; las cuales son dos. Sin embargo la ecuacin o x3 = 8 slo tiene una solucin en R que es x = 3 8 = 2 que es una sola. En los o o nmeros complejos la situacin es ms estable. En general, si a = 0 la ecuacin u o a o xn = a

3. Representacin geomtrica o e

111 n a no es

tiene exactamente n soluciones en C. Es decir, si a = 0, el s mbolo un slo nmero, sino n. o u Propiedad 22. | cos + i sin | = 1

Teorema 18 (ra ces de un n mero complejo). Si a = r(cos + i sin ) C u y n N entonces las races n-simas de a son e + 2k + 2k k = n r cos + i sin n n con k = 0, 1, 2, . . . , n 1. Demostracin. Se quiere encontrar los C tales que o n = a = r(cos + i sin ). (40) Escribimos en forma trigonomtrica: e = R(cos + i sin ) con R = ||. Entonces la ecuacin (40), con la ayuda de la frmula de o o Moivre, se transforma en Rn (cos n + i sin n) = r(cos + i sin ) (41)

tomando mdulos a ambos lados de sta ecuacin y en vista de la propiedad o e o 22 queda Rn = r. Siendo una ecuacin en R con n meros positivos, se obtiene que o u R = n r. Cancelando r a ambos lados de (41), cos n + i sin n = cos + i sin lo que implica cos n = cos sin n = sin lo que a su vez implica que n = + 2k para alg n k Z. Despejando u = + 2k n (42)

112

5. N meros complejos u

por lo que debe de tener la forma = + 2k + 2k n + i sin r cos n n (43)

pero no todos los n meros de la forma de la ecuacin (43) son diferentes: u o pongamos + 2k + 2k k = n r cos + i sin n n entonces n = + 2 + i sin n = n r(cos + i sin ) n n = 0 ; n r cos + 2 n

de forma similar, n+1 = 1 , n+2 = 2 , etctera. Por lo tanto, e = n r cos + 2k + 2k + i sin n n

para alg n k = 0, . . . , n 1. u

Ejemplo 81. Resolver la ecuacin o x4 = 4 Sol. El n mero x tiene que ser una ra cuarta de 4. Se pone 4 en u z forma trigonomtrica e 4 = 4(cos + i sin ) entonces x= + 2k + 2k 4 4 cos + i sin 4 4 4 4 cos + i sin 4 4 1 1 = 2 + i 2 2 = 1+i

con k = 0, 1, 2, 3. Por lo que x=

3. Representacin geomtrica o e

113

o bien x= + 2 + 2 2 cos + i sin 4 4 1 1 = 2 + i 2 2 = 1 + i x= 5 5 2 cos + i sin 4 4 = 1 i 7 7 2 cos + i sin 4 4 = 1 i.

o x=

En resumen x = 1 + i 1 + i 1 i 1 i. Ejemplo 82. Encontrar las ra ces c bicas de 8i u Sol. En forma trigonomtrica e 8i = 8 cos pues
T

3 3 + i sin 2 2

3 2

por lo que las ra ces buscadas son 3/2 + 2k 3/2 + 2k 3 + i sin k = 8 cos 3 3

8i

114

5. N meros complejos u

es decir 0 = 2 cos 3 3 + sin 6 6 = 2 cos + i sin 2 2 = 2i

1 = 2 cos

3/2 + 2 3/2 + 2 + i sin 3 3 7 7 + i sin = 2 cos 6 6 1 sin = , 6 2 3 cos = 6 2

Se sabe que

entonces 1 = 2 (cos + i sin ) cos + i sin 6 6 3 1 +i = 2 2 2 = 3i Adems a 2 = 2 cos 3/2 + 4 3/2 + 4 + sin 3 3 11 11 + i sin = 2 cos 6 6 = 2 cos 2 + i sin 2 6 6 = 2 (cos 2 + i sin 2) cos + i sin 6 6 = 2 cos i sin 6 6 = 3 i.

Por lo tanto, las ra ces c bicas de 8i son u 0 = 2i, w1 = 3 i, w2 = 3 i.


Maxima

solve(x^3=-8*%i,x);
TEX Output

3. Representacin geomtrica o e

115

x = (1) 6 (1) 6

1 1 1 3i, x = (1) 6 3i + (1) 6 , x = 2 (1) 6

Tarea 31. (1) Resolver las siguientes ecuaciones (2) Calcular las races cuartas de 1 i 3 (3) Calcular 2i. x4 = 16i, x4 = 1 + i

Cap tulo 6

Polinomios

Una expresin de la forma o f (x) = an xn + an1 xn1 + . . . + a1 x + a0 donde a0 , a1 , . . . , an son constantes y x variable, se llama polinomio en x. Los nmeros a0 , a1 , . . . , an se llaman coecientes, los trminos a n xn , an1 xn1 , . . . , a1 x, u e a0 se llaman monomios. Si an = 0 entonces f (x) se dice de grado n que se denota n = f (x) y en tal caso an se llama coeciente principal. Obsrvese e que el s mbolo 0 no est denido. a El monomio a0 se llama trmino independiente. e Ejemplo 83. Los siguientes son polinomios f (x) = 3x3 x + 2, g(x) = x4 x2 + 2x h(x) = 2x2 + (3 + 2x + 4) El polinomio f (x) tiene grado 3, coeciente principal 3 y trmino ine denpendiente 2; g(x) tiene grado 4, coeciente principal 1 y trmino ine e denpendiente 0; h(x) tiene coeenciente principal 2, grado 2 y trmino independiente 4.

Ejemplo 84. Las funciones f (x) = no son polinomios. 1 + 2, x h(x) = x+1 , x1 g(x) = x4 + x 1

117

118

6. Polinomios

1. Algebra
Los polinomios se pueden sumar, restar y multiplicar siguiendo las leyes asociativas, conmutativas y distributivas. Ejemplo 85. Multiplicar x3 x + 1 por x2 + 1. Sol. Tenemos que, distribuyendo (x3 x + 1)(x2 + 1) = x5 + x3 x3 x + x2 + 1 = x5 + x2 x + 1

Es mejor separar los coecientes: Ejemplo 86. Multiplicar f (x) = x5 + x3 2x2 + 3 por g(x) = 2x4 3x3 + 4x2 1. Sol. Separamos los coecientes en una tabla: f (x) : g(x) : 1 0 2 1 2 0 3 3 4 0 1 2 0 3 0 0 12

es decir

4 3 0 2 0 2 2 3 6

0 0 3 4 7

1 0 1 0 0 0 4 8 0 6 0 9 0 6 9 2 10

14 0 3

f (x)g(x) = 2x9 3x8 + 6x7 7x6 + 9x5 2x4 10x3 + 14x2 3 El mismo ejercicio en Maxima:
Maxima

(x^5+x^3-2*x^2+3)*(2*x^4-3*x^3+4*x^2-1);
TEX Output

2x4 3x3 + 4x2 1

x5 + x3 2x2 + 3

Maxima

2. Algoritmo de la divisin o

119

expand(%);
TEX Output

2x9 3x8 + 6x7 7x6 + 9x5 2x4 10x3 + 14x2 3

Tarea 32. Multiplicar por coecientes separados (1) x4 + x3 + x2 + x + 1 por x4 x3 + x2 x + 1 (2) 2x4 3x3 + x 1 por x4 + 3x2 1 (3) x4 + 4x3 5x2 2 por x4 4x3 5x2 2

2. Algoritmo de la divisin o
Los nmeros enteros Z y los polinomios tienen una estructura muy parecida. u As como hay un algoritmo de divisin en Z, lo hay en los polinomios. o Teorema 19 (algoritmo de la divisin). Si f (x), g(x) son polinomios, eno tonces existen polinomios unicos q(x) y r(x) tales que f (x) = g(x)q(x) + r(x) donde r(x) < g(x) o r(x) = 0. La forma usual en que se coloca la ecuacin del algoritmo de la divisin es: o o

q(x) g(x) f (x) r(x)

donde f (x) se llama dividiendo, g(x) se llama divisor, q(x) es el cociente, y r(x) es el residuo. La forma de obtener el cociente y el residuo la da el algoritmo largo de la divisin, que se ejemplica enseguida. Tambin hacemos uso de coecientes o e separados.

120

6. Polinomios

Ejemplo 87. Dividir x8 + x7 + 3x4 1 por x4 3x3 + 4x + 1 Sol. 1 3 0 4 1 1 1 0 0 1 3 0 4 4 0 4 4 12 0 12 4 12 36 32 32 1 4 12 32 82 3 0 0 0 1 1 2 0 16 4 14 4 0 0 48 12 14 52 12 0 96 0 128 32 82 52 140 32 1 82 246 0 328 82 192 140 360 83

es decir, el cociente es x4 + 4x3 + 12x2 + 32x + 82 y el residuo es 192x3 140x2 360x 83 o equivalentemente x8 + x7 + 3x4 1 = (x4 3x3 + 4x + 1)(x4 + 4x3 + 12x2 + 32x + 82) + 192x3 140x2 360x 83.

En Maxima la instruccin para obtener el cociente es quotient(f,g,x) o que produce el cociente de f (x) por g(x) en variable x:
Maxima

quotient(x^8+x^7+3*x^4-1,x^4-3*x^3+4*x+1,x);
TEX Output

x4 + 4x3 + 12x2 + 32x + 82

Similarmente remainder produce el residuo:


Maxima

remainder(x^8+x^7+3*x^4-1,x^4-3*x^3+4*x+1,x);
TEX Output

194x3 140x2 360x 83

2. Algoritmo de la divisin o

121

Tarea 33. Dividir por el mtodo de coecientes separados e (1) x7 + 3x6 2x3 + 3x2 x + 1 por x4 x + 1 (2) x5 3x2 + 6x 1 por x2 + x + 1 (3) x1 0 + x5 + 1 por x2 + x + 1

(4) 2x7 3x6 + x5 3x4 + 5x3 4x2 + 2x 1 por 2x3 3x2 + x 1

2.1. Regla de Runi. Cuando se divide por polinomios de la forma x c el residuo tiene que ser de grado menor o igual a (x c) = 1, es decir, el residuo tiene que ser una constante. Tal residuo constante y el correspondiente cociente son fciles de obtener empleando el mtodo conocido como regla de a e Runi. Enseguida justicamos y explicamos tal mtodo. e Supongamos que tenemos el problema de dividir el polinomio f (x) por xc donde f (x) = an xn + an1 xn1 + + a1 x + a0 entonces se debe de cumplir f (x) = q(x)(x c) + r donde r es una constante y q(x) es el cociente. Pongamos q(x) = bn1 xn1 + bn2 xn2 + + b1 x + b0 entonces, de (44), an xn + an1 xn1 + + a1 x + a0 (44)

= (bn1 xn1 + bn2 xn2 + + b1 x + b0 )(x c) + r = bn1 xn + bn2 xn1 + + b0 x+ cbn1 xn1 cb1 x cb0 + r

= bn1 xn + (bn2 cbn1 )xn1 + + (b0 cb1 )x cb0 + r igualando coecientes an = bn1 an1 = bn2 cbn1 an2 = bn3 cbn2 a1 = b0 cb1 a0 = cb0 + r . . .

122

6. Polinomios

despejando bn1 = an bn2 = an1 + cbn1 bn3 = an2 + cbn2 . . . b0 = a1 + cb1 r = a 0 + c 0 b0 es conveniente poner stas ecuaciones en una tabla: e c) an an1 an2 cbn1 cbn2 bn2 bn3 a1 a0 cb1 c0 a0 b0 r

bn1

Ejemplo 88. Encontrar el cociente y el resto de la divisin de 3x 6 7x5 + o 5x4 x2 6x 8 por x + 2. Sol. Ponemos 2) 3 7 6 8 5 0 1 6 26 62 124 246 504

es decir, el cociente es y el residuo es

3 13 31 62 123 252 496

q(x) = 3x5 13x4 + 31x3 62x2 + 123x 252 r = 496.

Ejemplo 89. Dividir 5x5 7x3 + 6x2 2x + 4 por x 1. Sol. Por la regla de Runi 1) 5 0 7 6 2 4 5 5 2 4 2 entonces el cociente es con residuo 5 5 2 4 2 6

q(x) = 5x4 + 5x3 2x2 + 4x + 2 r = 6.

3. Divisibilidad

123

Tarea 34. Calcular el cociente y el resto de dividir (2) x4 + 7x3 4x2 por x 3 (1) 2x4 6x3 + 7x2 5x + 1 por x + 2 (3) 10x3 2x2 + 3x 1 por 2x 3

(4) (n 1)xn nxn1 + 1 por (x 1)2

3. Divisibilidad
Si no interesa el cociente de la divisin por xc se puede calcular exclusivamente o el resto sin siquiera hacer uso de la regla de Runi. Teorema 20 (del resto). Si f (x) es un polinomio, entonces el residuo de la divisin de f (x) por x c es f (c). o Demostracin. Por el algoritmo de la divisin o o donde r es constante, entonces f (x) = (x c)q(x) + r f (c) = (c c)q(c) + r

es decir f (c) = r que es el residuo.

Precisamente el cociente no es tan importante como el residuo en cuestiones de divisibilidad. Denicin 90. Sean f (x), g(x) polinomios. Se dice que f (x) es divisible o por g(x) si al dividir f (x) por g(x) resulta residuo cero. Ejemplo 91. Demostrar que f (x) = x3 + x2 5x + 3 es divisible por x + 3. Sol. El residuo de la divisin de f (x) por x + 3 es f (3), seg n el o u teorema del resto. Pero f (3) = (3)3 + (3)2 5(3) + 3 = 27 + 9 + 15 + 3 = 0.

Por lo tanto f (x) es divisible por x + 3. Ejemplo 92. Demostrar que f (x) = xn cn es divisible por x c.

Sol. Por el teorema del resto, el residuo de la divisin de f (x) por x c o es f (c) = cn cn = 0. Luego f (x) es divisible por x c.

124

6. Polinomios

Tarea 35. Sin efectuar la divisin demostrar que o (1) x4 + 3x3 + 3x2 + 3x + 2 es divisible por x + 2 (2) x5 3x4 + x2 2x 3 es divisible por x 3

Propiedad 23. Si a, b son nmeros diferentes y f (x) es divisible por xa y u adems f (x) es divisible por x b entonces f (x) es divisible por el producto a (x a)(x b). Demostracin. Por hiptesis o o f (x) = q(x)(x a) (45)

pero como f (x) tambin es divisible por x b entonces el residuo de la e divisin de f (x) por x b debe de ser f (b) = 0. Luego de (45), o 0 = f (b) = q(b)(b a) es decir q(b)(ba) = 0, pero como ba = 0, se debe de obtener que g(b) = 0, que es el residuo al dividir g(x) por x b, es decir, existe q 1 (x) polinomio tal que q(x) = (x b)q1 (x). Sustituyendo en (45) se obtiene que f (x) = (x a)(x b)q1 (x) es decir f (x) es divisible por (x a)(x b).

Ejemplo 93. Sin efectuar la divisin, demostrar que f (x) = 2x 6 + 2x5 + o x4 + 2x3 + x2 + 2 es divisible por x2 + 1. Sol. Tenemos que x2 + 1 = x2 i2 = (x i)(x + i). Ahora, el residuo de dividir f (x) por x i es f (i) = 2i6 + 2i5 + i4 + 2i3 + i2 + 2 = 2(i2 )3 + 2i4 i + i4 + 2i2 i + i2 + 2 = 2(1)3 + 2i + 1 2i 1 + 2 =0 lo que indica, seg n el teorema del resto que f (x) es divisible por x i. u (46)

4. Mximo com n divisor a u

125

Adems, el residuo de la divisin de f (x) por x + i es a o f (i) = 2(i)6 + 2(i)5 + (i)4 + 2(i)3 + (i)2 + 2 = 2i6 2i5 + i4 + 2i + i2 + 2 = 0. = 2(1)3 + 2i + i + 2i 1 + 2

lo que implica que f (x) es divisible por x + i. Luego seg n la propiedad 23, f (x) es divisible por (x+i)(xi) = x 2 +1. u

Tarea 36. Sin efectuar la divisin demostrar que o (1) 2x4 7x3 2x2 + 13x + 6 es divisible por x2 5x + 6 (2) x6 + 4x5 + 3x4 + 2x3 + x + 1 es divisible por x2 + x + 1

4. Mximo comn divisor a u


Denicin 94. Si f (x) y g(x) son polinomios divisibles por h(x), entonces o h(x) se llama divisor comn. u

Ejemplo 95. Sean g(x) = x6 + 2x5 + x3 + 3x2 + 3x + 2 = (x + 1)(x + 2)(x4 x3 + x2 + 1) entonces f (x) y g(x) tienen divisores comunes a (x + 1), (x + 2), , (x + 1)(x + 2) = x2 + 3x + 2 f (x) = x4 + 4x3 + 4x2 x 2 = (x + 1)(x + 2)(x2 + x + 1)

Denicin 96. Si f (x), g(x) son polinomios entonces su mximo comn o a u divisor es el divisor comn de mayor grado. u Ejemplo 97. El mximo com n divisor entre x 4 + 4x3 + 4x2 x 2 y a u 6 + 2x5 + x3 + 3x2 + 3x + 2 es x2 + 3x + 2, seg n el ejemplo inmediato x u anterior. El mximo comn divisor se puede calcular en Maxima con gcd(f,g,x). a u
Maxima
Initial

gcd(x^6+2*x^5+x^3+3*x^2+3*x+2,x^2+3*x+2,x^4+4*x^3+4*x^2-x-2,x);
TEX Output

126

6. Polinomios

x2 + 3x + 2

El mximo comn divisor entre dos polinomios f (x), g(x) se puede calcular a u con el algoritmo de Euclides: supongamos que f (x) g(x), entonces q1 (x) g(x) f (x) r1 (x) q2 (x) r1 (x) g(x) r2 (x) q3 (x) r2 (x) r1 (x) r3 (x) . . . qk+1 (x) rk (x) rk1 (x) rk+1 (x) mximo comn divisor a u qk+2 (x) rk+1 (x) rk (x) 0 Es fcil mostrar que si c = 0 es una constante y h(x) es divisor comn de a u f (x) y g(x) entonces ch(x) es tambin un divisor comn. De sta observacin e u e o se har uso para evitar los quebrados en el clculo del mximo comn divisor. a a a u Ejemplo 98. Encontrar el mximo com n divisor de a u f (x) = x6 + 2x5 + x3 + 3x2 + 3x + 2 g(x) = x4 + 4x3 + 4x2 x 2

4. Mximo com n divisor a u

127

Sol. Por el algoritmo de Euclides, dividimos f (x) por g(x) utilizando coecientes separados: 1 4 4 1 2 1 1 1 2 4 2 0 1 3 3 2 4 4 1 2 2 4 2 5 3 2 8 8 2 4 4 10 3 1 2 4 16 16 4 8 6 13 3 10 1/6 1 12 1 11 6 1 12 6 6 72 10 4 72 1 11 1 12 6 6 72 10 4 12 6 24 72 33 110 57 38 /19 3 2

enseguida deber amos poner 6 13 3 10 1 4 4

pero para evitar quebrados multiplicamos por 6 el dividendo: 6 13 3 10 1 4 4 6 24 24 6 13 3 11 27

pero de nuevo, para evitar quebrados, multiplicamos por 6 6 13 3 10 1 4 4 6 24 24 6 13 3 11 27 66 162 66 143 19 1

donde el residuo ha sido dividido entre 19. Continuamos con una nueva divisin: o 6 5 1 3 2 6 13 3 10 6 18 12 5 15 10 5 15 10 0 0 0 entonces el mximo com n divisor es x 2 + 3x + 2. a u

128

6. Polinomios

Maxima

gcd(x^6+2*x^5+x^3+3*x^2+3*x+2,x^4+4*x^3+4*x^2-x-2,x);
TEX Output

x2 + 3x + 2

Ejemplo 99. Encontrar el mximo com n divisor de a u f = x5 x4 2x3 + 2x2 + x 1, Sol. Por el algoritmo de Euclides: 5 4 6 4 1 1 1 1 1 2 2 1 1 5 5 5 10 10 5 5 5 4 6 4 1 1 4 6 4 5 5 5 20 30 20 25 5 4 6 4 1 24 24 24 24 /24 1 1 1 1 5 4 6 4 1 5 5 5 5 1 1 1 1 1 1 1 1 0 0 0 0 g = 5x4 4x3 6x2 + 4x + 1

1 1 1 1

por lo tanto, el mximo com n divisor es a u x3 x 2 x + 1 Tarea 37. Encontrar el mximo comn divisor de los siguientes polinomios: a u (1) f = 2x4 + 2x3 3x2 2x + 1, g = x3 + 2x2 + 2x + 1 (2) f = x4 6x2 8x 3, g = x3 3x 2 (3) f = 2x5 + 4x4 + x3 x2 + x + 1, g = 6x5 2x4 + x3 + 2x2 x + 1 (4) f = 2x6 + 3x5 + x4 + 7x3 + 4x2 + 4x + 5, g = x4 x3 x 1

5. El teorema fundamental del algebra

129

5. El teorema fundamental del lgebra a


Uno de los problemas ms comunes que involucran a polinomios es el clculo a a de sus ra ces. Denicin 100. Sea f (x) un polinomio. o (1) Un nmero c C se dice raz o cero si u f (c) = 0. f (x) = 0 es una raz de f (x). (3) Resolver f (x) = 0 signica encontrar las races de f (x). El calcular una ra de un polinomio es equivalente a factorizarlo. z Teorema 21. Sea f (x) un polinomio. c es una raz de f (x) q(x) tal que f (x) = (x c)q(x). Demostracin. (): Si c es ra de f (x) entonces f (c) = 0, lo cual implica, o z seg n el teorema del resto que el residuo de f (x) al dividir por x c es cero. u Luego, por denicin existe q(x) polinomio tal que f (x) = (x c)q(x). o (2) Una solucin de o

(): Si f (x) = (x c)q(x) entonces f (c) = (c c)q(c) = 0.

Propiedad 24. Si f (x), g(x) son polinomios de grado n y m respectivamente, entonces (2) ( f (x)g(x) ) = n + m (1) ( f (x) + g(x) ) max{n, m}

Demostracin. Supongamos que o g(x) = bm xm + bm1 xm1 + + b1 x + b0 (1) Si n > m entonces luego f (x) = an xn + + am xm + + a1 x + a0 f (x) = an xn + an1 xn1 + + a1 x + a0

f (x) + g(x) = an xn + + (am + bm )xm + + (a1 + b1 )x + a0 + b0

de donde ( f (x) + g(x) ) = n = max{n, m}. De forma anloga se a obtiene el mismo resultado si n < m.

130

6. Polinomios

Si n = m entonces f (x) + g(x) = (an + bm )xn + + (a1 + b1 )x + (a0 + b0 ) dependiendo de si an +bm = 0 or no se obtiene que ( f (x)+g(x) ) = n o ( f (x) + g(x) ) < n es decir ( f (x) + g(x) ) n = max{n, m}. En cualquier caso: ( f (x) + g(x) ) max{n, m}. (2) Tenemos que f (x)g(x) = an bm xn+m + (an1 bm + an bm1 )xn+m1 + + a0 b0 y como an = 0 y bm = 0 entonces an bm = 0, lo que implica que ( f (x)g(x) ) = n + m.

Teorema 22. Si f (x) es polinomio de grado n entonces f (x) tiene a lo ms a n races. Demostracin. Si c1 , . . . , ck son ra de f (x) diferentes entre s entonces o ces , f (x) es divisible por x c1 , f (x) es divisible por x c2 , . . . , f (x) es divisible por x ck . Aplicando la propiedad 23 se obtiene que f (x) es divisible por (x c1 )(x c2 ) . . . (x ck ), de donde se obtiene que existe q(x) polinomio tal que f (x) = (x c1 ) . . . (x ck )q(x) entonces n = f (x) = k + q(x), por propiedad 24 po lo que n n g(x) = k = # ra ces

Ejemplo 101. Resolver la ecuacin o x2 + 1 = 0 Sol. Tenemos que i2 + 1 = 0 y tambin e (i)2 + 1 = 0

5. El teorema fundamental del algebra

131

por lo que i, i son soluciones a la ecuacin propuesta. Pero a n ms, son o u a las unicas pues no puede haber ms de dos, seg n el teorema inmediato a u anterior.

El hecho de ste ejemplo puede generalizarse. e Propiedad 25. Sea a R con a > 0, entonces la ecuacin o x2 + a = 0 tiene como unicas soluciones a x = i a, o en otros smbolos: x = i a

a = i a

Demostracin. Tenemos que o 2 (i a)2 + a = i2 a + a = 1(a) + a =0 y 2 (i a)2 + a = i2 a + a =0 o es decir, i a son soluciones de la ecuacin; y de nuevo, por el teorema inmediato anterior, stas son las unicas. e = 1(a) + a

Una de las deniciones ms empleada en los nmeos complejos es a u Denicin 102. o i=

Ejemplo 103. Resolver x2 + x + 1 = 0

132

6. Polinomios

Sol. Por la frmula general para las ra o ces de las ecuaciones de grado dos: 1 1 4 x= 2 1 3 = 2 1 1 3 = 2 1 3 = i 2 2 En general, el clculo de ra de polinomios es un problema def (aunque a ces cil existen mtodos para su calculo aproximado, como por ejemplo el mtodo de e e Newton-Ralphson). El problema se simplica si se conocen algunas ra ces. Ejemplo 104. Resolver la ecuacin curtica o a x4 5x2 10x 6 = 0 conociendo las ra ces: 1 y 3. Sol. Se debe de cumplir x4 5x2 10x 6 = (x + 1)(x 3)q(x) y lo que falta es encontrar las ra ces de q(x). Haremos sto primero calcue lando q(x) usando la regla de Runi dos veces; ntese que o x4 5x2 10x 6 = q(x). (x + 1)(x 3) 1) 1 0 5 10 6 1 1 4 6 6 6 0 0

3) 1 1 4 3 6 1 Por lo tanto 2 2

f (x) = (x + 1)(x 3)(x2 + 2x + 2). Ahora resolvemos x2 + 2x + 2 = 0

5. El teorema fundamental del algebra 48 x= 2 2 4 = 2 = 1 i 2 Por lo tanto todas las ra ces son 1, 3, 1 + i, 1 i En particular x4 5x2 10x 6 = (x + 1)(x 3)(x 1 i)(x 1 + i) Tarea 38. (1) Escribir una ecuacin cbica con races o u (a) 0, 1, 2. (b) 1, 1 + i, 1 i (3) Resolver

133

(47)

(2) Escribir una ecuacin curtica con races i, i, 1 + i, 1 i. o a 20x3 30x2 + 12x 1 = 0

siendo 1/2 una raz. (4) Resolver

2x4 x3 17x2 + 15x + 9 = 0 si 1 + 2 y 1 2 son races. Resulta que siempre existen ra de polinomios con coecientes complejos. ces Teorema 23 (fundamental del algebra). Sea f (x) un polinomio con coe cientes complejos no constante, entonces la ecuacin o f (x) = 0 tiene al menos una solucin en C. o Una demostracin del teorema fundamental del algebra puede verse en [2] o o en [3]. Ejemplo 105. La ecuacin o x2 + 1 = 0 es imposible de resolver en R, sin embargo en C tiene dos soluciones: i y i

134

6. Polinomios

Resulta que cualquier polinomio admite una factorizacin como la de (47) o si se aplica te teorema fundamental del algebra varias veces. Denicin 106. Si f (x) es un plinomio tal que f (x) = 1 entonces f (x) o se llama polinonio lineal. Corolario 4. Si f (x) es polinomio con coecientes complejos y no constante entonces existen 1 , . . . , n C tales que f (x) = an (x 1 ) . . . (x n ) (48) Demostracin. Por el teorema fundamental del algebra y el teorema del o resto, existe 1 C y f1 (x) polinomio tales que f (x) = (x 1 )f1 (x) (49) donde f1 (x) = n 1 y el coeciente principal de f 1 (x) es an . Si de nuevo aplicamos el teorema fundamental del algebra y el teorema del resto, ahora a f1 (x) obtenemos que deben de existir 2 C y f2 (x) polinomio tales que f1 (x) = (x 2 )f2 (x) (50) donde f2 = n 2 y el coecente principal de f2 (x) es an . Etctera: existen e n C y fn1 (x) polinomila tales que fn1 (x) = (x n )fn (x) (51) tales que fn (x) = 0 y fn (x) tiene coeciente principal an : necesariamente fn (x) = an . Sustituyendo (50),. . . ,(51) en (49) se obtiene que f (x) = (x 1 ) . . . (x n )an

Los nmeros 1 , . . . , n de la factorizacin (48) de f (x) son precisamente u o las ra ces de f (x). No todas ellas son diferentes; pueden repetirse. Pordr a 2 2x + 1, ocurrir, por ejemplo, para f (x) = x f (x) = (x 1)(x 1). Si se agrupan las ra ces iguales en (48), entonces se obtiene una factorizacin: o (52) f (x) = (x 1 )m1 (x 2 )m2 . . . (x k )mk donde, ahora s los nmeros i son diferentes entre s , u . Denicin 107. El nmero mi en la factorizacin (52) se llama la multio u o plicidad de la raz i 1 i k.

5. El teorema fundamental del algebra

135

Ejemplo 108. La ecuacin o x2 + 2x + 1 = 0 tiene como soluciones, seg n la frmula general a u o 2 4 4 x= = 1 2 que es una sola ra pero tiene multiplicidad 2 porque z, x2 + 2x + 1 = (x + 1)2

Propiedad 26. El polinomio f (x) se factoriza como f (x) = (x )m g(x) con g() = 0 es raz de f (x) de multiplicidad m. Denicin 109. o (1) (a) 1 entonces (b) 2 entonces (c) 3 entonces Si es una raz de multiplicidad se llama raz simple; se llama raz doble; se llama raz triple

Si es raz de multiplicidad > 1 entonces se llama raz mltiple. u

Denicin 110. Si n N; o f (n) (x) =

dn f (x) dxn

Lema 3. Si es raz de f (x) de multiplicidad m entonces existen polinomios h1 (x), . . . , hm (x) tales que f (1) (x) = (x )m1 h1 (x) h1 () = 0 h2 () = 0 f (2) (x) = (x )m2 h2 (x) . . .

f (m) (x) = hm (x)

hm () = 0

Demostracin. Tenemos que existe g(x) polinomio tal que o f (x) = (x )g(x), con g(x) = 0.

136

6. Polinomios

entonces, derivando f (1) (x) = m(x )m1 g(x) + (x )m g(x)


h1 (x)

= (x )m1 mg(x) + (x )g (x))

donde h1 (x) = mg() = 0. Derivando de nuevo, como f (1) (x) = (x )h1 (x), f (2) (x) = (x )m2 (m 1)h1 (x) + (x )h1 (x))
h2 (x)

etctera. e Teorema 24. Si es raz de f (x) tal que f (1) () = 0, f (2) () = 0, . . . , f (m1) () = 0, f (m) () = 0 entonces m es la multiplicidad de . Demostracin. Sea k la multiplicidad de . Tenemos que demostrar que o k = m. Seg n el lema anterior, f (1) () = 0, . . . , f (k1) () = 0, pero u (k) () = 0. Asi que k = m contradecir la hiptesis. f a o Ejemplo 111. Sea f (x) = x2 + 2x + 1. Sabemos que 1 es ra de f (x). z Adems como a f (1) (x) = 2x + 2, f (2) (x) = 2, f (1) (1) = 2 + 2 = 0; f (2) (1) = 2 = 0,

lo que implica que 1 es ra de multiplicidad 2. Lo cual es claro, pues, z f (x) = (x + 1)2 .

Ejemplo 112. Sea n N con n > 1 y f (x) = x n nx + n 1. Probar que x = 1 es ra y encontrar su multiplicidad. z Sol. Tenemos que f (1) = 1 n + n 1 = 0 por lo que 1 es ra Calculemos su multiplicidad: z. f (2) (x) = n(n 1)xn2 , f (1) (x) = nxn1 n, f (2) (1) = n(n 1) = 0, pues n > 1, f (1) (1) = n n = 0,

lo que implca que 1 es ra de f (x) de multiplicidad 2. z

6. Acotacin de ra o ces

137

Que en el ejemplo anterior, 1 es ra de f (x) = x n nx + n 1 de z multiplicidad 2 signica que existe un polinomio g(x) tal que f (x) = (x 1)2 g(x).

por lo tanto

Se puede calcular el polinomio g(x): debido a que g(x) = f (x)/(x 1) 2 , podemos hacer tal clculo dividiendo g(x) entre x 1 dos veces usando la regla a de Runi. 1) 1 0 0 0 0 n n 1 1 1 1 1 1 1n 1) 1 1 1 1 1 1n 0 1 2 3 n 2 n 1 1 2 3 4 n 1 0

xn nx + n 1 = (x 1)2 (xn1 + 2xn2 + 3xn3 + + (n 2)x + n 1). Tarea 39. (1) Escribir el polinomio de menor grado f (x) tal que tiene a 1 y a 1 como races simples, 2 como raz doble y 3 como raz cudruple. a (2) Descomponer en factores lineales: (a) x3 1 (b) x4 1 (c) x4 + x2 + 1 (d) x4 x2 + 1 (e) x4 + 1

6. Acotacin de ra o ces
En general, el clculo exacto de ra de polinomios es un problema dif a ces cil. Sin embargo se pueden dar rangos entre los cuales se encuentran las ra ces reales, si las hay. Denicin 113. Sea A R conjunto. o (1) Un nmero K R se llama cota superior de A si u a A, K a (2) Un nmero k R se llama cota inferior si u a A, k a .

138

6. Polinomios

Ejemplo 114. Sea A = {1, 4, 2, 7, 8}. Una cota superior de A es 10 mientras que una cota inferior de A es 4. Tales cotas no son las unicas; tambin 1000 es cota superior y 100000 es cota inferior. e Es fcil ver que si un conjunto tiene una cota superior (inferior) entonces a tiene una innidad de ellas. 6.1. Acotacin de ra o ces positivas. Sea f (x) = an xn + an1 xn1 + + a1 x + a0 con an , . . . , a0 R. Sea c R positivo cualquiera y dividimos f (x) por x c segn la regla de Runi: u an b0 = a n es decir, f (x) = (x c)(b0 xn1 + b1 xn2 + + bn1 ) + rn . (53) an1 an2 cb0 cb1 b1 b2 a1 a0 cbn2 cbn1 bn1 rn

Entonces, si b0 0, b1 0, . . . bn1 0 pero rn > 0, se obtiene de (53), para x c que f (x) = (x c)( b0 xn1 + b1 xn2 + + bn1 ) + rn >0 entonces x (c, ) f (x) > 0 . Por lo tanto, si hay ra de f (x) stas no pueden estar en [c, ), por lo tanto ces e deben de estar en (, c). Se concluye entonces que c es cota superior de las ra ces. Lo anterior nos da un algoritmo para acotar las ra ces positivas. Se debe de tomar c > 0 tal que al dividir por Runi, todos los coecientes resulten positivos o cero, y el ultimo (residuo) positivo. Noese que el coeciente b1 t debe de cumplir b1 = can + an1 0; lo cual da una pista sobre c, se debe de tomar tal que an c + an1 0. Adems an 0. a Ejemplo 115. Hallar una cota superior de las ra positivas de la ecuacin ces o 2x5 7x4 5x3 + 6x2 + 3x 10 = 0
0 0 0 0 0 0 >0

6. Acotacin de ra o ces

139

Sol. Notemos que el coeciente principal en 2 > 0. Tomamos c tal que 2c 7 0. Luego proponemos c = 4: 4) 2 7 8 2 1 5 6 3 10 4 1 <0

como 1 < 0 entonces c = 4 es muy peque o para ser cota superior; debe n intentarse con un n mero ms grande, por ejemplo c = 5: u a 5) 2 7 5 6 3 10 10 15 50 280 1415 2 3 10 56 283 1405 Por lo tanto c = 5 es cota superior de las ra ces positivas.

En general, si f (x) es un polinomio de coecientes reales entonces las ra ces de f (x) corresponden a las intersecciones de la grca de f (x) con el eje x: a

0 Fun1 5000

5000

2000

5
For bautista Wed Apr 28 12:24:04 PDT 2004

Figura 1. La grca de f (x) = 2x5 7x4 5x3 + 6x2 + 3x 10 a

Ejemplo 116. Hallar una cota superior para las ra ces positivas de x5 7x4 100x3 1000x2 + 10x 50 = 0

140

6. Polinomios

Sol. Se propone c tal que c 7 > 0. Por ejemplo c = 8, 8) 1 7 100 1000 10 50 8 8 1 1 92 proponemos c = 10 10) 1 7 100 1000 10 50 10 30 1 3 70 ahora c = 20 20) 1 7 100 1000 10 50 20 260 3200 44000 1 13 160 2200 44010 43960 entonces c = 20 es cota superior de las ra ces positivas.

7. Acotacin de ra o ces negativas


Si se desea encontrar una cota inferior para las ra negativas de ces f (x) = an xn + an1 xn1 + + a1 x + a0 = 0 se hace el cambio de variable y = x: f (y) = an (1)n y n + an1 y n1 + + (1)a1 y + a0 = 0

y se procede como antes, en la variable y (si a n (1)n < 0 se multiplica por 1) para encontrar las cotas superiores de las ra ces positivas de f (y) que denotamos con c, y entonces c es la cota inferior buscada. Ejemplo 117. Hallar una cota inferior para las ra ces negativas de la ecuacin o 2x6 + 20x5 + 30x3 + 50x + 1 = 0. Sol. Se hace x = y:

2y 6 20y 5 30y 3 + 50x + 1 = 0

se propone c tal que 2c 20 0, por ejemplo c = 10: 10) 2 20 0 30 0 50 1 20 0 0 2 0 0 30

8. Ra ces enteras

141

se propone entonces c = 11: 11) 2 20 0 30 0 50 1 22 22 242 2332 2 2 22 212 2332 ? > 0 ? > 030 Entonces 11 es una cota inferior para las ra negativas. ces Tarea 40. Hallar cotas inferiores y superiores para las races de (1) x4 7x3 + 10x2 30 = 0 (2) x4 8x3 + 12x2 + 16x 50 = 0

(4) x5 + x4 + x2 25x 100 = 0

(3) x5 + 8x4 14x3 53x2 + 56x 18 = 0

8. Ra ces enteras
Recordemos la relacin de divisibilidad en los enteros Z. o Denicin 118. Sean a, b Z. Se dice que a es divisor de b si al dividir b o por a resulta residuo cero; o equivalentemente si q Z tal que b = qa. Ejemplo 119. El n mero 2 es divisor de 8, pero 8 no es divisor de 2; u tambin 2 es divisor de 8. e Propiedad 27. Si f (x) = an xn + an1 xn1 + + a0 es un polinomio con coecientes enteros, entonces las races enteras de f (x) son divisores de a0 . Demostracin. Sea c Z ra de f (x). Entonces o z despejando a0 :

an cn + an1 cn1 + a1 c + a0 = 0

factorizando c del lado derecho:

a0 = an cn an1 cn1 a1 c a0 = c (an cn1 a1 )

lo que implica que c es un divisor de a 0 .

142

6. Polinomios

Ejemplo 120. Hallar todas las ra ces enteras de x 2 + 1 = 0. Sol. Las ra ces enteras de f (x) = x2 + 1 deben de ser divisores de 1, sto es 1 o 1. Pero f (1) = 2 = 0 y f (1) = 2 = 0. Por lo tanto no existen e ra ces enteras de x2 + 1 = 0.

Ejemplo 121. Averiguar si la ecuacin o x6 + 3x5 36x4 45x3 + 93x2 + 132x + 140 = 0 tiene ra ces enteras o no, y calcularlas en caso armativo. Sol. Las ra ces enteras deben ser divisores de 140; pero 140 = 22 7 5 por lo que las ra ces enteras deben estar entre la siguiente lista: 1, 2, 4, 5, 7, 10, 14, . . . , 140 y para encontrar cules son ra a ces, deber amos calcular f (1), f (2), f (4), f (5), f (7), f (10), f (14), . . . , f (140) y desechar los que no son cero. Para ahorrar clculos es mejor acotar las a ra ces. Acotacin de ra o ces positivas: debemos tomar c 0 tal que c + 3 > 0, i.e, c 3. Elegimos c = 1: 1) 1 3 36 45 93 132 140 1 4 1 4 32 falla. Probamos con un n mero ms grande que 1: c = 4, u a 4) 1 3 36 45 93 132 140 1 28 1 7 8 falla de nuevo. Ahora c = 6: 6) 1 3 36 45 93 132 140 6 54 108 378 1 9 18 63 > 0 > 0 > 0 por lo que c = 6 es cota superior de las ra ces positivas. Acotacin de ra o ces negativas: hacemos el cambio de variable x = y para obtener: y 6 3y 5 36y 4 + 45y 3 + 93y 2 132y + 140 = 0 (54)

8. Ra ces enteras

143

debemos elegir c tal que c 3 0, por ejmplo c = 3:

falla. Ponemos c = 6,

3) 1 3 36 45 93 132 140 3 0 1 0 36 6) 1 3 36 45 93 132 140 6 18 1 3 18

falla de nuevo. Proponemos c = 8:

8) 1 3 36 45 93 132 140 8 40 32 1 5 4 77 > 0 >0 >0 Por lo tanto c = 8 es una cota inferior para las ra negativas. Es decir, ces las ra ces reales deben estar entre 8 y 6. Por lo que de la lista (54) slo o consideramos a 1, 2, 4, 5, 7 Checaremos si cada una es ra usando Runi y el teorema del resto. Primero z c = 1; 1) 1 3 36 45 93 132 140 1 4 32 77 16 148 1 4 32 77 16 148 288 = f (1) = 0 por lo que 1 no es ra Ahora c = 1; z. 1) 1 3 36 45 93 132 140 1 2 38 7 100 32 1 2 38 7 100 32 108 = f (11) = 0

por lo que 1 tampoco es ra z.

es decir 2 es ra Podemos ahora proceder con c = 2 de la misma forma: z. 2) 1 3 36 45 93 132 140 2 2 76 62 62 140 1 1 38 31 31 70 0 = f (2)

Ahora con c = 2: 2) 1 3 36 45 93 132 140 2 10 52 194 202 140 1 5 26 97 101 70 0 = f (2)

(55)

y obtener que 2 es tambin ra Pero es mejor si nos auxiliamos de (55), e z. porque de (55) tenemos que f (x) = (x 2)(x5 + 5x4 26x3 97x2 101x 70)

144

6. Polinomios

y ahora debemos de tratar de factorizar el factor x 5 + 5x4 26x3 97x2 101x 70 por x + 2 y repetir el proceso para cada ra Es decir z. 3 36 45 93 132 140 2 10 52 194 202 140 2) 1 5 26 97 101 70 0 = f (2) 2 6 64 66 70 1 3 32 33 35 0 = f (2) por lo que 2 es ra Intentamos con 5, z. 5) 1 3 32 33 35 5 40 40 35 1 8 8 7 0 = f (5) entonces 5 es ra Enseguida con c = 5; z. 5) 1 8 8 7 5 15 35 1 3 7 42 = f (5) = 0 8 8 7 7 7 7 1 1 1 0 = f (7) 2) 1

Finalmente con c = 7:

7) 1

Por lo tanto, las ra enteras de la ecuacin son ces o 2, 2, 7 . Tarea 41. Hallar las races enteras de (2) x3 9x2 + 22x 24 = 0 (1) x3 2x2 25x + 50 = 0

(4) x4 x3 x2 + 19x 42 = 0 Ejemplo 122. Hallar las ra ces enteras de x5 + x4 20x3 44x2 21x 45 = 0 Sol. Las ra ces enteras son los divisores de 45: 45 = 32 5 es decir las posibles ra ces enteras son: 1, 3, 5, 9, 15, 45 enseguida acotamos las ra ces.

(3) x3 106x 420 = 0

8. Ra ces enteras

145

Positivas: se debe tomar c + 1 0, i.e., c 1, por ejemplo c = 1: 1) 1 1 20 44 21 45 1 2 18 1 2 18

6) 1 1 20 44 21 45 6 42 132 408 1 7 22 78 > 0 > 0 por lo que c = 6 es cota superior de las ra ces. Negativas: se pone x = y: multiplicamos por 1: y 5 + y 4 + 20y 3 44y 2 + 21y 45 = 0 y 5 y 4 20y 3 + 44y 2 21y + 45 = 0 1) 1 1 20 44 21 45 1 0 1 0 20

debemos tomar c 1 0, por ejemplo c = 1;

Por lo tanto las ra ces reales estn entre 6 y 6. Por lo que las posible a ra ces enteras son 1, 3, 5. 1) 1 1 20 44 21 45 1 2 18 62 83 1 2 18 62 83 123 = f (1) = 0 1) 1 1 no es ra z. 1 20 44 21 45 1 0 20 22 1 1 0 20 22 1 46 = f (1) = 0

6) 1 1 20 44 21 45 6 30 60 600 > 0 1 5 10 100 > 0 > 0 por lo que 6 es una cota inferior de las ra ces negativas.

4) 1 1 20 44 21 45 4 12 1 03 8

1 no es ra z.

146

6. Polinomios

3 no es ra z. 3) 1

3) 1 1 20 44 21 45 3 12 24 204 ? 1 4 8 68 225 f (3) = 0

Por lo tanto, las ra ces enteras son:

1 20 44 21 45 3 6 42 6 45 5) 1 2 14 2 15 0 = f (3) 5 15 5 15 5) 1 3 1 3 0 = f (5) 5 10 55 1 2 11 f (5) = 0 3, 5.

9. Ra ces racionales
El mtodo para encontrar ra en Q se basa en el siguiente teorema. e ces Teorema 25. Si c Q es una raz de

f (x) = xn + an1 xn1 + + a1 x + a0

donde cada coeciente es entero, entonces c Z. Demostracin. Podemos poner c = a/b con a, b Z sin factores comunes o no triviales. Entonces a n a n1 a 0 = f (a/b) = + an1 + + x + a0 b b b multiplicando a ambos lados de sta ecuacin por b n : e o despejando an : an + an1 an1 b + + a1 abn1 + bn a0 = 0 an = b (an1 an1 a1 abn2 bn1 a0 ) lo que implica que y s tienen factores comunes y entonces r y s tienen factores comunes no triviales . Lo cual es imposible a menos que s = 1. Y entonces c = r/1 = r Z. El teorema puede usarse para trartar con situaciones como la que sigue: rn

9. Ra ces racionales

147

Dado: f (x) = an xn + an1 xn1 + + a1 x + a0 polinomio con coecientes racionales. Problema: encontrar las ra racionales de f (x). ces El algoritmo que da la solucin comienza con hacer un cambio de variable: o y x= k donde k es un nmero entero a determinar de tal forma que u kn f (y/k) an resulte un polinomio (en y) con coecientes enteros. Debido al teorema anterior las ra racionales de ste son enteras, por lo que podemos aplicar losmetodos ces e de acotacin de ra o ces enteras para obtener las soluciones enteras en y. Una vez logrado tal, hay que regresar a variable x. Ejemplo 123. Encontrar las soluciones racionales de 6x4 7x3 + 8x2 7x + 2 = 0 . Sol. Se hace x = y/k: y4 y3 y2 y 7 3 +8 2 7 +2 = 0 4 k k k k multiplicando por k 4 /6: 7 8 7 2 y 4 ky 3 + y 2 k 2 k 3 y + y = 0 6 6 6 6 se puede tomar k = 6 para que todos los coecientes resulten enteros: 6 Buscamos ra ces racionales de sta, tales ra e ces son mas bien enteras, por el teorema inmediato anterior (el coeciente principal es 1 y los dems con a enteros). Por lo que las ra ces buscadas son divisores de 432 = 2 4 33 acotemos tales ra ces. Primero las ra positivas: tomamos c tal que c7 ces 0; c = 7, 7) 1 7 48 252 432 7 0 336 > 0 1 0 48 > 0 > 0 Por la tanto 7 es cota superior de las ra ces. Ahora las ra ces negativas; hacemos y = z, en (56), para obtener: z 4 + 7z 3 + 48z 2 + 252z + 432 = 0 y buscamos una cota superior para las ra positivas de sta (puede notarse, ces e debido a que los coecientes resultantes son todos ellos positivos, que tal 1, 2, 4, 6, 8, . . . , 432 g(y) = y 4 7y 3 + 48y 2 252y + 432 = 0 . (56)

(57)

148

6. Polinomios

ecuacin no tiene ra o ces positivas). ejemplo c = 1, 1) 1 7 7 1 14

Buscamos c > 0 tal que 6c + 7 0, por 48 252 432 14 62 > 0 62 > 0 > 0

Hemos obtenido que las ra ces enteras estn entre 1 y 7 inclusive. De a la lista (57) obtenemos que las posible ra ces son 1, 2, 4, 6. probamos con cada una de stas usando la regla de Runi: e 1) 1 7 48 252 432 1 6 54 306 1 6 54 306 738 = g(1) por lo que 1 no es ra z, 1) 1 7 48 252 432 7 0 48 204 1 0 48 204 g(1) = 0 2) 1 7 48 252 432 2 10 76 352 1 5 38 176 80 = g(2) 3) 1 7 48 252 432 3 12 108 432 4) 1 4 36 144 0 = g(3) 4 0 144 6) 1 0 36 0 = g(4) 6 36 1 6 72 = g(6) por lo tanto las unicas ra ces racionales de (56) son: y = 3 y y = 4. Regresemos ahora a la variable x, recordando que x = y/6 resulta que las ra ces pedidas son: x= 1 3 = , 6 2 x= 4 2 = 6 3

por lo tanto c = 1 es cota inferior de las ra ces negativas.

Ejemplo 124. Hallar las ra ces racionales de la ecuacin o 25x4 70x3 126x2 + 414x 243 = 0 .

9. Ra ces racionales

149

Sol. Hacemos el cambio x = y/k, para obtener 25 y4 y3 y2 y 70 3 126 2 + 414 243 = 0 4 k k k k

multiplicamos por k 4 /25, 243 4 70 3 126 2 2 414 3 ky k y + k y k =0 25 25 25 25 se elige k = 5 resultando y4 g(y) = y 4 14y 3 126y 2 + 2070y 6075 = 0 1, 3, 5, 9, 15, . . . , 6075 . (58)

que tiene por ra ces racionales a enteros divisores de 6075 = 3 5 52 : (59) Acotamos las ra ces positivas de (58): tomamos c > 0 tal que c 14 0. Tomamos c = 14: 14) 1 14 126 2070 6075 14 0 108 1 0 126 20) 1 14 126 2070 6075 20 120 1 432 1 6 6 21) 1 14 126 2070 6075 21 147 441 52731 1 7 21 2511 46656 por lo tanto c = 21 es cota superior de las ra ces. Ahora hacemos y = z: z 4 + 14z 3 126z 2 2070z 6075 = 0 1) 1 14 126 2070 6075 1 15 1 15 111 15) 1 14 126 2070 6075 15 435 4635 1 29 309 2565 >0 Por lo tanto las ra ces enteras estn entre 15 y 21 inclusive. De (59) a se sigue que las posibles ra ces enteras son 1, 3, 5, 9, 15 por lo que c = 15 es cota inferior para las ra ces. y tomamos c > 0 tal que c + 14 0; c = 1;

150

6. Polinomios

Por Runi, se desechan 1, 3. Probemos 5:

tambin se desecha 9 y 15. Chequemos 15: e

5) 1 14 126 2070 6075 5 45 855 6075 1 9 171 1215 0 = g(5) 15) 1 9 171 1215 15 90 1215 1 6 81 0 = g(15)

por lo tanto la ra ces racionales de (58) son y = 5 y y = 15. Regresando a variable x = y/5 queda que las ra ces racionales buscadas son 5 15 x = = 1, x = =3 . 5 5 Tarea 42. Hallar las races racionales de las siguientes ecuaciones (2) 2x3 + 12x2 + 13x + 15 = 0 (3) 6x4 11x3 x2 14 = 0 (4) 4x4 11x2 + 9x 2 = 0 (5) 6x5 + x4 14x3 + 4x2 + 5x 2 = 0 (6) 2x4 4x3 + 3x2 5x 2 = 0 (1) 3x3 26x2 + 34x 12 = 0

Bibliograf a

[1] H. Cdenas, E. Lluis, F. Raggi y F. Toms, Algebra Superior, Trillas, Mxico, 1973. a a e [2] A. G. Kurosch, Curso de Algebra Superior, Limusa, Mxico, 1994. e [3] J.V. Uspenski, Teor de Ecuaciones, Limusa, Mxico, 1993. a e

151

Indice

Argumento 87 Binomial 10 Caso base 6 Cero, de un polinomio 107 Cociente 99 Coeciente principal 97 Coecientes separados multiplicacin 97 o divisin 99 o Complejo puro 80 Conjugado 86 Cota inferior superior demoivre 90 Determinante 47 Desarrollo por menores 59 Divisible 102 Divisor comn 104 u Eje imaginario 87 Eje real 87 Euclides, algoritmo de 104 Euler, identidad de 89 Fibonacci, nmeros de 14 u Forma binmica 80 o Forma trigonomtrica 88 e Forma polar 88 Gauss, mtodo de 26 e Gauss-Jordan, mtodo de 30 e gcd 104 Grado 97 Hipercaja 58 Lineal, polinomio Matriz 17 adjunta 73 aumentada 26

cero 22 de coecientes 26 entrada 17 escalonada 31 Identidad 19 invertible 70 inversa 70 menor 57 orden 17 registro 17 tamao 17 n transpuesta 60 triangular 27 superior 27 inferior 27 Moivre, frmula de 92 o Mdulo 86 o Monomios 97 Multiplicacin por escalares 37 o Multiplicidad Newton-Ralphson, mtodo de 109 e Norma 41 Operaciones elementales 27 Parte imaginaria 86 Parte real 86 Pascal, tringulo de 10 a Plano complejo 87 Polar, forma 88 polarform 89 Principio de induccin matemtica 5 o a generalizado 14 Producto cruz 52 quotient 99 Ra de un polinomio 107 z remainder 100 Resto, teorema del 102

153

154

Indice

Residuo 99 Runi, regla de 100 Sistemas de ecuaciones cuadrados 31 Solucin 107 o Submatriz 69 Suma de anglos 91 u Sumatoria 11 Sustitucin regresiva 25 o Trmino independiente 97 e Tringulo de Pascal 11 a Trigonomtrica, forma e Variables libres 33 Vectores la 37 columna 37

Das könnte Ihnen auch gefallen